4120 1344
Exam Time
NOTEPAD
Results
of 130 questions answered correctly

You have reached of 130 points, ( %)

Your time

Question 1 of 130

1. During examination of a patient, the doctor detected in him disorders of the eyes (hemeralopia, Bitots spots), skin and skin appendages, mucosa, and gastrointestinal tract. He was provisionly diagnosed with Prasad\'s syndrome. What causes the development of this pathology?

Explanation

Prasad Syndrome is a condition characterised by geophagia ( the habit of eating soil, clay etc), hypogonadism, growth retardation and zinc deficiency. Bitot spots are keratin fragments found/deposited on the conjunctiva; they are usually found due to Vitamin A deficiency. Zinc deficiency may also be involved with the pathogenesis of secondary vitamin A deficiency. Inadequate zinc can depress the hepatic synthesis of retinol-binding protein (RBP), which is required for mobilization of retinol from the liver. In addition, zinc may play a role in the conversion of beta-carotene to retinol via the enzyme 15-15 dioxygenase. 

 

In iron deficiency, iron deficiency anemia will be observed ( brittle nails, soreness in the tongue, weakness, pale skin etc). Copper deficiency is seen in Menkes disease.

2. A 45-year-old man with thrombophlebitis of the deep veins in his legs suddenly after physical exertion developed sharp pain in his thorax on the right, dyspnea, and hemoptysis. Objectively his condition is severe; he presents with acrocyanosis, shortening of pulmonary percussion^ sound on the right, and weakened respiration. Respiration is 30/min., blood pressure is 110/80 mm Hg. ECG shows sinus tachycardia, his heart rate is 120/min., the electrical axis of the heart deviates to the right, S7-Q777. What is the most likely diagnosis?

Explanation

Notice that this patient has thrombophlebitis- a condition in which blood clots form and block  an already inflamed vein(s). This condition can be superficial or deep ( deep vein thrombosis- as in this case). Pulmonary embolism is one of the various complications of DVT- this occurs when a clot or substance formed elsewhere travels and blocks an artery in the lungs. Sharp pain in the thorax and hemoptysis coupled with weakened respiration are clinical findings in this condition.
3. A 37-year-old woman received an occupational trauma that resulted in a severe vision impairment. Now she needs to be trained for another occupation. What type of rehabilitation should the doctor choose for the patient in this case?

Explanation

Occupational/ professional Rehabilitation involves specialized healthcare dedicated to improving, maintaining and restoring the physical strength, cognition and mobility with maximum results. It helps an individual to recover from addictions, injuries and disorders.

Social rehabilitation — a set of activities aimed at rehabilitation and improving the functional capability of people and their inclusion in the society.

 

Psychological rehabilitation focuses on restoring one’s mental status.

4. A 22-year-old postparturient woman on the 12th day after the normal childbirth informs of fever up to 39°(7 for the last 3 days and pain in her right mammary gland. The right mammary gland is enlarged, hot to touch, tense, hyperemic, and painful. Palpation reveals there a dense infiltration 8x8 cm with a fluctuation in its center. What is the most likely diagnosis?

Explanation

From the question stem, it is evident that she’s on the 12th day post-partum. On examination of the right mammary gland, there are clear signs of inflammation (mastitis) which is infiltrative with a fluctuation (Infiltrative-purulent) 

 

Mastitis is a bacterial infection of the mammary gland. It occurs mostly in pregnant or postpartum women; due to lactation, the nipples present with some erosions and are enlarged making it easier for bacterial invasion. This infection is often characterised by the production of purulent exudates.

5. A 38-year-old woman has been suffering from glomerulonephritis for 20 years. For approximately 16 years she has been presenting with progressing renal parenchymal arterial hypertension that became refractory and accompanied by leg edemas. She receives a combination of 100 mg losartan and 20 mg lercanidipine with insufficient antihypertensive effect. What medicine can she be recommended for intensification of the antihypertensive effect of her therapy?

Explanation

Notice that this patient has Progressing renal parenchyma arterial hypertension accompanied by edemas of the leg; she is already on a combined medication that includes Losartan ( Angiotensin receptor blocker) and Lercanidipine ( a calcium channel blocker). On the basis that this patient has suffered from glomerulonephritis, currently having a renal induced hypertension, presents with edema and does not have a diuretic prescribed as part of the complex treatment, the prefered drug of choice to intensify this treatment will be Torasemide ( a loop diuretic).

This patient is already on an Angiotensin receptor blocker, there is no need to use an ACE inhibitor ( Lisinopril);  we need to try another class with a different mechanism of action.

6. A 45-year-old man has been suffering from ankylosing spondylitis for 15 years. For the last 3 years he has been noticing facial swelling and edemas of the limbs. Objectively he assumes a ’’beggar’s” position. X-ray shows,’bamboo spine” changes in the thoracic and lumbar segrilents. Heart ultrasound shows aortic regurgitation. Complete blood count: Hb- 106 g/L; leukocytes - 8.9 ■ 1010 /L; ESR- 40 mm/hour. Daily proteinuria - 9.6 grams per 24 hours. Blood creatinine ⼔ 230 mcmol/L. What is the cause of kidney failure in this case?

Explanation

 

Ankylosing spondylitis is a chronic inflammatory disease of the axial skeleton that leads to partial or even complete fusion and rigidity of the spine. The most common renal  complication of ankylosing spondylitis is Renal amyloidosis. Amyloidosis is a collective term for the extracellular deposition of abnormal proteins, either in a single organ (localized amyloidosis) or throughout the body (systemic amyloidosis). The different subtypes of amyloidosis are categorized according to the origin of the deposited proteins (e.g., AA, AL). Renal amyloidosis is characterised by nephrotic syndrome; in this patient we can see a high level of proteinuria coupled with facial swelling and edema ( due to decreased oncotic pressure).

7. A patient was brought into the pulmonology department with complaints of inspiratory dyspnea and dry cough at the highest point of inhalation. On examination the following is observed: pale skin, cyanotic lips, ’Hippocratic fingers? Auscultation detects Velcro-type crackles (like opening a Velcro fastener). X-ray shows a ’ground glass opacity” pattern. What is the most likely diagnosis?

Explanation

Hippocratic nails refer to a condition in which there is proliferation of the distal part of the fingers, especially the nail bed. Velcro Type crackles refer to bilateral crepitation heard especially during slow inspiration but can be associated with expiratory crackles. These types of crackles are seen in fibrotic interstitial lung diseases.

 

Hand-schuller-christain disease is characterised by a triad of single or multiple punched out bone lesions on the skull, uni/bilateral exophthalmos and diabetes insipidus. Other findings in this disorder include; infantilism, dwarfism and polyuria.

8. A 45-year-old man came to the hematologist with complaints of general weakness, elevated body temperature, excessive sweating, enlarged cervical lymph nodes. Objectively his body temperature is 37.5°C, the skin is pale and dry the posterior cervical lymph nodes are dense and elastic, up to 2 cm in diameter, mobile. There are no peculiarities in the patient\'s heart and lungs. Hepatosplenomegaly was detected. What examination is necessary to determine the scope of the pathologic process?

Explanation

Of the above listed diagnostic procedures, the Computed tomography will be most appropriate in determining the scope of the pathologic process. Note that on examination, enlarged cervical lymph nodes were found; a CT scan is the best procedure ( among the listed) in examining these lymph nodes. An ultrasound will be appropriate for diagnosis of pathologies relating to internal organs.
9. A 65-year-old man underwent a left hemicolectomy due to a malignant tumor in the descending colon. On the 4th day after the surgery he developed pain and edema in his left shin. The Homans sign is positive on the left. What postoperative complication developed in this patient?

Explanation

A positive Homan’s sign ( calf pain at the dorsiflexion of the foot ) indicates a case of Deep Vein Thrombosis (blood clot within a deep vein). Patients with DVT are mostly predisposed by (Virchow Triad)

Stasis - due to post operation (as in this case), long drive/flight.

Hypercoagulability- can be due to a defect in coagulation cascade proteins eg factor V Leiden; oral contraceptive use.

Endothelial damage. 

One major complication of DVT is pulmonary embolism.

 

Unfractionated heparin or LMWH are used for prophylaxis or acute management.

10. A 35-year-old forestry officer was delivered to the hospital on the 7th day after the onset of the disease. He complains of chills, elevated body temperature up to 40.0°C, sharp headache, and myalgias. On examination his face is puffy and hyperemic, the tongue is dry, \"chalk- dusted.\" In the left inguinal area, a sharply painful conglomeration of enlarged lymph nodes can be palpated. The skin over the conglomeration is hyperemic and tense. What etiotropic therapy should be prescribed to this patient?

Explanation

11. Examination detected vesicles with seropurulent content on the neck, back of the head, and buttocks of an infant on the 4th day of life. The patients condition is satisfactory the child is active, all newborn reflexes can be fully induced, the umbilical cord is at the stage of mummification, the umbilical area is without any peculiarities. What disease can be suspected?

Explanation

Vesiculopustulosis is a purulent inflammation found in  newborns. Their causative agents include staphylococci, streptococci, e.coli etc. Clinical features – Evanescent vesicles (a few mm in diameter —> first transparent, later muddy content —> break down over 2-3 days —> small erosions, then crusts which disappear without pigmentation. Localization - skin of buttocks, hips, inguinal folders, head. Treatment is usually local – chlorhexidine, fucorcin, brilliant green.

Pemphigus neonatorum is characterised by the presence of erythematous patches coupled with the presence of vesicles and small bullaes usually localised in the lower abdomen and on extremities. The bullaes later form crusts but pigmentation is absent. 

 

Miliaria is also called sweat rashes, in this case we see numerous very itchy rashes caused due to the blockage or clogging of sweat glands.

12. A 65-year-old man with acute anterior myocardial infarction developed an asphyxia attack. Examination detected diffuse cyanosis. In the lungs there are numerous heterogeneous wet crackles. Heart rate is 100/min. Blood pressure is 120/100 mm Hg. What complication occurred in this patient?

Explanation

Note that the key finding here are the numerous wet crackles heard from the lungs; these crackles are heard when there is excess fluid in the airways. Pulmonary edema is usually seen in heart related issues ( in this case post MI); in this condition air sacs in the lungs are filled with fluid making it difficult to breathe. 

Cardiogenic shock can also be caused by Myocardial Infarction but its clinical features include; Tachycardia, Hypotension, Cold/clammy extremities, weak pulse, slow capillary refill etc. 

 

The blood pressure of this patient is 120/100 mmHg which rules out the option of hypertensive crisis, a condition in which the blood pressure is equal or greater than 180/120mmHg.

13. A 13-year-old girl for a month has been complaining of fatigability dull pain in her right subcostal area, abdominal distension, and constipations. Abdominal palpation reveals positive Kehr, Murphy and Ortner signs, while Desjardins and Mayo-Robson points are painless. Ibtal bilirubin is 14.7 mcmol/L, predominantly indirect, ALT- 20 U/L, AST- 40 U/L, amylase - 6.3 mmol/L. Echocholecystography shows practically no contraction of the gallbladder. Make the provisional diagnosis:

Explanation

Kehr’s sign- pain in the left shoulder ( due to irritation of the under surface of the diaphragm).

Murphy’s sign- pain during inspiration while palpating the right subcostal region.

Ortner’s sign-  painfulness at the easy pushing on the right costal arch by the edge of the palm. 

 

These signs are positive in conditions related to the gallbladder eg cholecystitis, cholelithiasis etc. The presence of a normal amylase level rules out the option of acute or chronic pancreatitis. The results from Echo Cholecystography shows that the gallbladder underwent NO  contraction thereby making the flow of bile (kinesis) restricted - this condition is referred to as a hypokinetic biliary dyskinesia.

14. A 48-year-old woman has been hospitalized due to development of tachysystolic atrial fibrillation. She has lost 5 kg of body weight within 2 months. On palpation there is a node in the left lobe of the thyroid gland. What pathology resulted in the development of this condition?

Explanation

The presence of tachysystolic atrial fibrillation and a sudden loss of weight indicates the presence of hyperthyroidism. The presence of nodes on palpation indicates a nodular goiter ( butterfly shaped in the mixed form). A toxic goiter is a form of goiter that causes hyperthyroidism while a non toxic goiter is one that does not cause any hyper/hypothyroidism.
15. A 58-year-old man complains of weakness and tumor-like formations that appeared on the anterior surface of his neck and in the inguinal region. Palpation detects soft painless mobile cervical and inguinal lymph nodes up to 2 cm in diameter. The liver protrudes by 2 cm from the edge of the costal margin, the lower splenic pole is at the umbilical level. In blood: erythrocytes -3.5 • 1012 /L, Hb- 88 g/L, leukocytes -86 ■ 109 /L, band neutrophils - 1%, segmented neutrophils -10%, lymphocytes -85%, eosinophils - 2%, basophils - 0%, monocytes-2%, erythrocyte sedimentation rate - 15 mm/hour, Gumprecht shadows. What is the most likely diagnosis?

Explanation

Note that from the blood analysis, the key finding is the presence of Gumphrect shadow (Smudge cells). These cells are seen in Chronic lymphocytic leukemia (CLL).

Chronic Myeloid leukemia (CML) is defined by Philadelphia Chromosome ( a translocation between chromosome 9 and 22) coupled with a myeloid stem cell proliferation.
16. A newborn has a round red formation in the suprapubic region. Examination shows that urine is being discharged in pulses from the two orifices located in the lower part of this formation. Name this developmental anomaly:

Explanation

Bladder Exstrophy

Bladder exstrophy is a rare developmental abnormality that is present at birth (congenital) in which the bladder and related structures are turned inside out. Exstrophy means 'turned inside out'.

It is an abnormality of formation of the bladder and the bony pelvis. The bladder does not form into its normal round shape but instead is flattened and exposed on the abdominal wall. The pelvic bones are also widely separated. The remainder of the lower urinary tract may also be flattened and exposed, with abnormal formation of the prostate and penis. 

 

Bladder exstrophy is a congenital abnormality that occurs when the skin over the lower abdominal wall (bottom part of the tummy) does not form properly. The bladder is open and exposed on the outside of the abdomen.

17. A 28-year-old man, a teacher, after an emotional stress developed painful muscle spasms in his right hand that occur during writing; now he has to hold the pen between the second and third fingers. He has no problems with typing or writing on the blackboard; no other motor disturbances or neurological pathologies are detected. What is the most likely diagnosis?

Explanation

Writer’s cramp is a specific form of focal dystonia that affects the fingers, hand, or forearm. Focal dystonia of the hands is a neurological movement disorder.

 

Cortical agraphia is a neurological disorder in which an individual loses the ability to communicate either through writing or he/she forgets how to spell.

18. A 45-year-old woman was hospitalized with complaints of periodical severe headaches against the background of elevated blood pressure up to 180/90 mm Hg, muscle weakness, and frequent urination (at night as well). Her anamnesis states that despite combining various antihypertensive drugs and adjusting their dosage her arterial hypertension cannot be corrected with drugs. The patient\'s blood serum potassium levels are 2.0 mmol/L, blood serum sodium levels are 160.0 mmol/L. Ultrasound imaging detects three- dimensional formations approximately 1.0 cm in diameter in the both adrenal glands. Selective endovascular blood sampling from the suprarenal veins was performed, which revealed significant increase of cortisol and aldosterone levels. Make the diagnosis:

Explanation

Notice that from ultrasound, this patient presents with a tumor of the adrenal gland. From the results of the blood and biochemical analysis, we see an elevated cortisol, aldosterone coupled with increase in sodium levels - this indicates that the tumor is present in the adrenal cortex ( ruling out the option of Pheochromocytoma which is a tumor of the adrenal medulla and is characterised by elevated catecholamines and blood pressure). This patient has an aldosteroma ( an aldosterone producing adenoma); recall that aldosterone is produced by the zona glomerulosa of the adrenal gland and is responsible for the resorption of sodium and water during formation of urine coupled with deposition of potassium.
19. A pregnant woman at 32 weeks of gestation with the risk of preterm labor undergoes a treatment to prevent fetal respiratory distress syndrome. What medicine was she prescribed?

Explanation

In  most premature babies, due to the deficiency of surfactant or hyaline membrane ( immature lungs), the baby has severe difficulty in breathing ( respiratory distress). For the prevention of this disorder, the mother is given glucocorticoids (steroid injection) before delivery to stimulate fetal lung maturation. Dexamethasone is a glucocorticoid. 

 

Misoprostol is a prostaglandin analogue indicated for stomach ulcers, to induce labor, stimulate abortion etc. Gynipral is a Beta 2 agonist used for the management of bronchial asthma.

20. A 45-year-old man underwent a cardiac surgery one week ago. His general state has been deteriorating since then: dyspnea at rest, retrosternal pain that irradiates to the neck, marked weakness. Objectively his body temperature is hectic. His cardiac borders are expanded, apical beat is weakened. Auscultation detects pericardial friction rub. What is the most likely diagnosis?

Explanation

A pericardial friction rub is highly specific for Acute pericarditis. Pericarditis is an inflammation of the pericardium. Acute pericarditis is most commonly caused by viral infection; however, a number of conditions can cause an inflammatory response in the pericardium. Acute inflammation typically manifests with fever, pleuritic chest pain, and a pericardial friction rub on auscultation. The diagnosis is established based on clinical findings, although diffuse ST segment elevations on ECG and imaging may support the diagnosis.
21. The director of a medical facility draws up a financial plan for the next year. To improve the economic well-being of his establishment, he decided to increase the amount of medical services provided. How will it change the fixed cost per unit of service?

Explanation

22. A woman with the pregnancy term of 8 weeks complains of elevated temperature up to 37.6°C, skin rash that can be characterized as macular exanthema, enlargement of posterior cervical and occipital lymph nodes, small amount of bloody discharge from the genital tracts. She was examined by the infectious diseases specialist and diagnosed with rubella. What tactics should the obstetrician-gynecologist choose?

Explanation

Rubella virus infection in early pregnancy has been frequently associated with adverse pregnancy outcomes due to its teratogenic effects. The infection is most common in children, young adults, child bearing aged women and pregnant women. Acute rubella virus infection in early pregnancy has been associated with poor pregnancy outcome ranging from spontaneous abortion, stillbirth and multiple birth defects known as Congenital Rubella Syndrome (CRS). CRS includes auditory, sensorineural, cardiac and ocular abnormalities.

 

The management of rubella infection depends on the gestational age at the onset of infection. 

 

• Infection before 18 Weeks of Gestation: The fetus is at high risk for infection and severe symptoms. Termination of pregnancy could be discussed and accepted, according to local legislation, particularly if the infection presented before 12 Weeks of Gestation. 

 

• Infection after 18 Weeks of Gestation: The pregnancy could be continued with simple ultrasound monitoring. A specific pediatric examination of the newborn and testing for RV-IgM are recommended.

 

Since this pregnancy is at 8weeks then the option of Abortion can be chosen to avoid giving birth to a child with multiple congenital defects - cataract, deafness, congenital heart disease.

23. A surgery unit received a person with an incised stab wound on the upper third of the right thigh. Examination detects an incised stab wound 3.0x0.5x2.0 cm in size on the inner surface of the upper third of the right thigh. Bright-red blood flows from deep within the woixnd in a pulsing stream. Characterize this type of bleeding:

Explanation

Note that the color of blood and the rate of its flow helps us detect the type of bleeding. Arterial bleeding is characterized by bright red color with a pulsating stream ( flow) while dark red blood is found in venous bleeding with the rate of flow being less.
24. A 23-year-old woman without visible cause developed a conflicting behavior at her workplace. She accused the management of underestimating her, claimed that she can be a deputy director, because she speaks four languages, is very attractive, and can make useful connections for the company She has been dressing extravagantly flirting with her colleagues, and singing loudly in her office. In fact, she has only the training of a computer operator and speaks no foreign languages. What is the likely clinical diagnosis?

Explanation

A manic episode is characterized by a sustained period of abnormally elevated or irritable mood, intense energy, racing thoughts, and other extreme and exaggerated behaviors. People can also experience psychosis, including hallucinations and delusions, which indicate a separation from reality. The symptoms of mania can last for a week or more and manic episodes may be spaced within periods of depression during which you may experience fatigue, sadness, and hopelessness.

 

Schizophrenia is mostly characterised by hallucinations, delusions, catatonic ( disorganised) behaviour and speech. 

25. A 45-year-old woman undergoes an inpatient treatment. She complains of elevated body temperature up to 39.0°C, pain in her right lumbar area, turbid urine with blood. CT scan shows an area of low density within the parenchyma, no difference between the cortical and medullary layers, and increased density of the perinephric fat due to edema. What is the diagnosis?

Explanation

26. A 25-year-old man was hospitalized with complaints of pain in his lower abdomen and right lumbar area that appeared one hour ago. The patienfs general state is moderately severe. Body temperature - 38.2°C, heart rate - 102/min. The tongue is dry The abdomen is painful on deep palpation in the right iliac area and in the Petit triangle. Aure-Rozanov and Gabay signs are positive. Make the provisional diagnosis:

Explanation

Aure-rozanov sign is commonly seen in cases of retrocecal appendix; it refers to the presence of pain in the right petit triangle when palpated.  The location of pain in the lower abdomen and right lumbar area rules out the option of cholecystitis ( right hypochondrium). The results from deep palpation of the abdomen helps put the provisional diagnosis of acute appendicitis.
27. A 56-year-old woman was diagnosed with stage 2 hypertension of the 2nd degree. She belongs to the group of moderate risk and has bronchial asthma. What group of drugs is CONTRAINDICATED to this patient?

Explanation

The groups of drugs mostly used in the management of hypertension include; ACE inhibitors, ARBs, calcium channel blockers , Beta blockers, diuretics etc. Recall that Beta-blockers are drugs that bind to beta-adrenoceptors and thereby block the binding of norepinephrine and epinephrine to these receptors. This inhibits normal sympathetic effects that act through these receptors. Beta receptors are not only found in the heart ( beta 2) but also in the lungs ( beta 1); blocking of these receptors will cause the heart to slow down but have increased force of contraction, in the lungs- Bronchoconstriction. Bronchoconstriction is very dangerous in a patient with asthma thus NON- SELECTIVE BETA BLOCKERS eg Atenolol are contraindicated.
28. A 68-year-old woman with congestive heart failure and left ventricular ejection fraction of <40.% receives the following pharmacotherapy scheme: ramipril, torasemide, bisoprolol, clopidogrel, and digoxin. During one of her regular examinations, frequent polymorphic ventricular extrasystoles were detected in the patient. What medicine should be removed from her therapy scheme?

Explanation

 

Although Digoxin helps in improving the ejection fraction in a patient with Heart failure, Notice that frequent polymorphic ventricular extrasystoles were detected. These polymorphic ventricular extrasystoles are usually caused by an elevation in calcium influx into myocardial cells. Recall that the mechanism of action of Digoxin involves the reversible inhibition of the Na-K ATPase enzyme, leading to various beneficial effects. The Na-K ATPase enzyme functions to maintain the intracellular environment by regulating the entry and exit of sodium, potassium, and calcium (indirectly). Na-K ATPase is also known as the sodium pump. The inhibition of the sodium pump by digoxin increases intracellular sodium and increases the calcium level in the myocardial cells, causing an increased contractile force of the heart. This improves the left ventricular ejection fraction (EF), an important measure of cardiac function. Therefore in order to stop these extrasystoles, Digoxin should be excluded from the therapy.

29. A 35-year-old woman had acute onset of the disease that started with fever up to 39.0°C and cough. 3 days later her dyspnea at rest increased up to 35/min. Downward from her right shoulder-blade angle, percussion detects a dull sound. No vocal fremitus, respiratory sounds cannot be auscultated. What is the treatment tactics?

Explanation

Pleural Effusion

For a patient with respiratory tract infection that manifests with fever, cough, tachypnea (35 cycles per min) and dyspnea at rest coupled with a dull percussion sound on percussion, this patient clearly has pleural effusion. This is a case of massive right pleural effusion that extends from the lower margin of the scapula (shoulder blade) to the lower lung margin. 

Common causes of Pleural Effusion include: cancer tumors; Pneumonia or other lung infection; Congestive heart failure; chronic lung diseases etc.

 

Pleural effusion is managed by performing a thoracentesis or Pleural tap. Thoracentesis, also known as a pleural tap, is a procedure done when there’s too much fluid in the pleural space. It could be diagnostic or therapeutic. 

Diagnostic: This allows a pleural fluid analysis to be performed in the lab to figure out the cause of fluid accumulation around one or both of the lungs. 

Therapeutic: Remove excess pleural fluid to assist respiration and provide symptomatic relief. This is what this patient needs to relieve the dyspnea. 

 

The pleural space is the small space between the lungs and the chest wall.

 

Take Note of these significant lung sounds: 

Dull percussion sounds - Consolidation (Pneumonia); Fluid (pleural effusion which could be hemothorax or pyothorax)

Crackles - Pleural Effusion

Wheeze - Bronchial Asthma

 

High resonance sounds - Pneumothorax

 
30. After playing with ’’mosaics’ : a 2- year-old child suddenly developed cough, stridorous respirations, urges to vomit, and cyanosis against the background of relative somatic health. What should the doctor suspect first when examining this child?

Explanation

Notice that from anamnesis, the child was playing with mosaics ( toys) just before the  described signs suddenly appeared; from this information, the doctor should suspect aspiration of the toys ( foreign bodies).
31. 40% of the workers, who polish the art glass, using an abrasive disk, and have a long record of employment, are diagnosed with ulnar neuritis, 21% - with vegetative polyneuritis, and 12% - with vegetomyofascitis of the upper limbs. These pathologies are associated with the following harmful factor:

Explanation

An abrasive disc is a heavy equipment used in cutting and drilling jobs; from anamnesis, these workers have been using these tools for a long period of time and currently present with issues related to the nerves and muscles. These conditions are mainly due to the effect of long term exposure to vibrations from this work tool and environment.
32. After a long drive with the window open a man developed a facial asymmetry; he cannot close his right eye, his right nasolabial fold is smoothed out, movements of expression are absent on the right, there is a disturbance of taste sensation in the tongue on the right. No other neurological abnormalities were detected. What disease can be provisionally diagnosed in this case?

Explanation

Facial (nerve) palsy is a neurological condition in which function of the facial nerve (cranial nerve VII) is partially or completely lost.  Key findings include; Sensory disturbances : Painful sensation around or behind the ear, Impairment of taste in the anterior tongue , Hyperacusis; dropping of the mouth, dry mouth, Bell's phenomenon: a physiologic, reflexive movement of the eye (upward and outward) that occurs when the eyelid is actively closed, Lagophthalmos: The patient cannot fully close the eyes (due to paralysis of the orbicularis oculi muscle). Decreased lacrimation. 

In neuropathy of the trigeminal nerve ( trigeminal neuralgia), the individual will feel excruciating pain at the slightest pressure applied on the face e.g. while brushing the teeth, yawning etc.

In neuropathy of the oculomotor nerve, the individual will experience diplopia and strabismus.
33. A 6-year-old girl complains of body temperature up to 39°C, rhinitis, dry cough, dyspnea. She has been presenting with these signs for 5 days already On examination her condition is of moderate severity Her dyspnea is of mixed genesis. Respirations are 28/min., pulse is 120/min. Percussion produces a dull sound in the right lower segments; in the same area auscultation detects weakened respiration and fine vesicular wet crackles; coarse respiration can be detected on the left. Make the provisional diagnosis:

Explanation

Take Note of these significant lung sounds: 

Dull percussion sounds - Consolidation (Pneumonia); Fluid (pleural effusion which could be hemothorax or pyothorax)

Crackles - Pleural Effusion

Wheeze - Bronchial Asthma

High resonance sounds - Pneumothorax

 

A young girl presented with upper respiratory tract infection and on percussion, a dull sound and wet crackles were heard. A normal lung will produce a resonance sound (high resonance - pneumothorax). Dull sound is often associated with Pneumonia which causes lung consolidation and can progress to form pleural effusion which produces the crackles.

Community-acquired pneumonia (CAP) refers to an acute infection of the pulmonary parenchyma acquired outside of the hospital. The main causative pathogens of Community-acquired pneumonia (CAP) are Streptococcus pneumoniae, influenza A, Mycoplasma pneumoniae and Chlamydophila pneumoniae. The clinical presentation of CAP varies, ranging from mild pneumonia characterized by fever and productive cough to severe pneumonia characterized by respiratory distress and sepsis.

 

Stenosing laryngotracheitis (Croup) - barking cough sound.

Acute bronchiolitis is more of an acute viral infection often associated with wheezing sound. 

 

Bronchitis affects the airways (Bronchi) and produces a large amount of mucus with cough. It can produce wheezing sound also. 

 
34. A full-term newborn (born with the body weight of 3900 g at gestational age of 39 weeks) on the first day of his life developed respiratory disturbances: dyspnea, arrhythmic respiration, cyanosis attacks. On examination there is* paradoxical respiration observed and left side of the chest lags behind in the act of breathing. On auscultation the respiration is weakened in the lungs on the left. Neurologist diagnosed the patient with leftsided Erb-Duchenne palsy Complete blood count shows no changes. What is the most likely diagnosis?

Explanation

Paradoxical respiration refers to movement of the chest walls in the opposite direction ie, chest walls move in when you inhale and move out when one exhales. Note that this patient has been diagnosed with Erb-Duchenne palsy ( waiter’s tip hand), a condition caused by the injury to nerve roots of C5 and C6. Recall that the  motor innervation of the diaphragm comes from the phrenic nerves C3-C5, this patient with Erb’s palsy has an injury to the C5 nerve therefore affecting the motor activity of the Diaphragm (a major muscle in respiration).
35. A 30-year-old woman made an appointment with the family doctor for scheduled vaccination of her 2-year-old child. What type of healthcare provides such medical services?

Explanation

According to levels of specialization, types of medical care include: Emergency, Primary, Secondary, Tertiary, palliative, medical rehabilitation and dentistry.

Primary medical care: involves care given by nurses, midwives, general physicians, family doctors, in polyclinics etc. One major aspect of primary health care is preventive medicine - these include vaccination, teaching on methods for good living etc.

Secondary medical care: When your primary care provider refers you to a specialist, you are then in secondary care. Secondary care simply means you will be taken care of by someone who has more specific expertise eg cardiologist, endocrinologist etc

Tertiary Medical Care: Once a patient is hospitalized and needs a higher level of specialty care within the hospital, he may be referred to tertiary care. Tertiary care requires highly specialized equipment and expertise.
36. A 3-year-old child has been brought to a hospital with complaints of pain in the legs, fever, and loss of appetite. Objectively: pale skin and mucosa, hemorrhagic rash. Lymph nodes are enlarged, painless, dense and elastic, not matted together. Bones, joints, and abdomen are painful. The liver and spleen are enlarged. Hemogram: Hb- 88 g/L, color index -1.3, platelets - 80 . 109 /L, leukocytes - 25.8 • 109 /L, lymphoblasts -70%, ESR- 52 mm/hour. Make the provisional diagnosis:

Explanation

This patient has an Acute lymphoblastic leukemia; Blast cells are immature WBC. Presence of 70% of Blast cells in the analysis indicates an acute form of leukemia. high numbers of lymphoblasts can also reduce the level of platelets , the cells that help blood clot (this patient has a platelet count of 80; the norm is 150-440). This can lead to increased bleeding; for example, you may have more frequent nosebleeds. Henoch-schonlein purpura ( hemorrhagic/ IgA vasculitis) is a condition in which small blood vessels are inflamed. It is an acute immune complex-mediated small vessel vasculitis that most commonly occurs in children. typically presents with a tetrad of symptoms: palpable purpura, arthritis/arthralgia, abdominal pain, and renal disease; although not all patients show the complete four symptoms
37. After semolina was introduced into the diet, a 1-year-old child for 2 months has been presenting with loss of appetite, irritability loss of body mass, and loss of previously learned skills. The feces are copious and foul-smelling. The skin is pale and dry, the hair is brittle. The abdomen is distended, while the limbs are thin. Stool test shows high levels of fatty acids. What is the most likely diagnosis?

Explanation

Notice that these symptoms begin to take place after the introduction of Semolina; a food product rich in gluten. Celiac disease, also referred to as celiac sprue or nontropical sprue, is a common condition characterized by a maladaptive immune response to gluten, a protein found in many grains (e.g., wheat). The underlying pathophysiology is believed to be a combination of gluten intolerance, which triggers an autoimmune reaction, and production of autoantibodies that target tissue transglutaminase, specifically within the proximal small intestine. Typical findings include changes in bowel habits and symptoms associated with malabsorption (e.g., fatigue, weight loss, vitamin deficiencies).

Irritable bowel syndrome is a functional disorder that affects mainly the large intestine and is characterized by bloating, abdominal cramps, diarrhea etc 

 

Mucoviscidosis refers to cystic fibrosis of the pancreas.

38. A 73-year-old woman came to the family physician for one of her regular follow-up examinations. Three months ago she was found to have type 2 diabetes mellitus. She was keeping to her diet and exercise plan and taking phytopreparations. On examination her fasting glucose was within the range of 78-8.6 mmol/L, HbAlc _ 79%. Height - 164 cm, weight - 83 kg. What blood glucose-controlling medicine should she be prescribed first in the course of her pharmacological therapy?

Explanation

 

This patient has been diagnosed with type 2 diabetes mellitus ( non-insulin dependent diabetes). The results from the current blood sugar level and glycated hemoglobin supports it ; Norm blood glucose 3.5-5.5 mmol/L; normal glycated hemoglobin -6 %. For the treatment of type 2 Diabetes, Metformin also known as Glucophage (a biguanide) is usually the first drug to be administered.

39. A patient, has gradually lost his consciousness. The skin is pale and dry There is smell of ammonia from the mouth. Respirations are deep and noisy Heart sounds are muffled, pericardial friction rub is present. Blood pressure is 180/130 mm Hg. Blood test: Hb- 80 g/L, leukocytes - 12 ■ 109 /L, blood glucose - 6.4 mmol/L, urea -50 mmol/L, creatinine - 1200 mcmol/L, blood osmolarity - 350 mOsmol/kg H2O. No urinary excretion. Make the diagnosis:

Explanation

 

Recall that Ammonia is converted to urea ( via the ornithine/urea cycle). Urea is then excreted from the body through the urine. Notice that this patient has no urinary output, smell of ammonia from the mouth coupled with an increased urea level in the blood. Recall that ammonia and urea (at high blood concentration like in this case) can cross the blood brain barrier leading to encephalopathy and further loss of consciousness. N/B the blood glucose and osmolarity levels are also elevated but not enough to lead to their respective forms of coma.

40. A 10-year-old boy came to the polyclinic with complaints of stuffy nose. It is known that these signs occur in the child periodically (in spring and autumn). He has a history of atopic dermatitis. The father of the child has bronchial asthma. Objectively the boy’s face is pale and slightly swollen. Respirations are 22/min. Auscultation detects vesicular respiration over the lungs. Rhinoscopy shows swollen and pale nasal mucosa. What disease can be suspected?

Explanation

This patient has a history of Atopic dermatitis and a family history of Atopic dermatitis; It is also stated that these signs usually arise in specific seasons- this helps us conclude that the reason behind the rhinitis is an allergy.

 

Allergic rhinitis is inflammation of the inside of the nose caused by an allergen, such as pollen, dust, mould or flakes of skin from certain animals.

41. Disease onset was acute. A child developed general weakness, pain in the joints, and fever Later these signs became accompanied by itching skin rash manifested as erythematous spots 2-5 mm in size. The rash gradually turned hemorrhagic. Large joints are painful and swollen; pain attacks periodically occur in the paraumbilical area; there are signs of intestinal hemorrhage. What is the most likely diagnosis?

Explanation

Henoch-schonlein purpura ( hemorrhagic/ IgA vasculitis) is a condition in which small blood vessels are inflamed. It is an acute immune complex-mediated small vessel vasculitis that most commonly occurs in children. typically presents with a tetrad of symptoms: palpable purpura, arthritis/arthralgia, abdominal pain, and renal disease; although not all patients show the complete four symptoms ( as in this case). Notice that this child presents with purpuras ( a rash that turns hemorrhagic), arthritis and abdominal pain- these help us put the diagnosis of Henoch-schonlein purpura. 

Streptococcal impetigo is characterized by papules that evolve into pustules then thick crust which are golden or honey-coloured. These lesions are usually found on the face and extremities. 

 

Scarlet fever also presents with hemorrhagic rash but arises on the background of a Streptococcal infection and also is characterised by a crimson red tongue.

42. A 34-year-old multipara was brought to the labor ward with regular labor activity. Her pelvic size is 26-29-32-22 cm. Vaginal examination shows 6 cm cervical dilation, the amniotic sac is unbroken. The fetus is in the breech presentation, with buttocks pressed to the entrance into the lesser pelvis. The promontory cannot be reached, no exostoses. Fetal heart rate is 140/min., expected fetal weight is 2800 g. What labor tactics should be chosen?

Explanation

The main external pelvic sizes: 

D. Spinarun -  distance between anterior superior iliac spines from both sides. It is 25-26 cm.

D. Cristarum – distance between iliac crista from both sides. It is 28-29 cm.

D. Trochanterica – distance between trochanter majors from both sides. It is 31-32 cm.

C. Externa -  distance between midpoint of superior surface of the symphysis pubis and suprasacralis fossa. It is 20-21cm.

 

In the question stem, the 34 year old woman is 26-29-32-22 cm. This values correspond to the normal values for the above listed main external pelvic sizes. Fetal heart rate - 140/min is normal (norm: 110 - 170 beats per minute). Fetal weight is also normal. Therefore, even though, the fetus is in breech presentation, the delivery can still progress through the natural birth canal.

43. A 54-year-old woman complains of a fogged vision in her right eye, rainbow circles in her vision, headache, and nausea. Within the last month she twice experienced a similar condition, but back then all the signs eventually disappeared and her sight was restored. Currently all the signs have been persisting for over 2 days. Objectively the patient has eyelid edema, congestive injection of the eyeball, corneal opacity shallow anterior chamber of the eye, and dilated pupil that is unresponsive to the light. Her intraocular pressure is 48 mm Hg. Make the diagnosis:,

Explanation

Observe that the intraocular pressure is 48mmHg ( norm is 10-21 mmHg). Glaucoma is a medical condition characterised by optic disc atrophy coupled with cupping, usually with an elevated intraocular pressure and progressive peripheral  visual field loss if untreated.

 

Iridocyclitis is an inflammation of the anterior uveal tract ( iris and ciliary body) ; symptoms include blurred vision, eyes pain, photosensitivity, red eye ( conjunctival injection) etc.

44. A 3-month-old child with signs of rickets presents with positive Chvostek, Trousseau, and Maslov signs. One day ago the parents witnessed a cyanotic attack in their child - the child broke into a cold sweat, the eyes bulged, and respiratory arrest occurred. One minute later the child drew in a loud breath and the child’s condition normalized again. What is the cause of the described signs of the disease?

Explanation

Chvostek sign:  twitching of the facial muscles in response to tapping over the area of the facial nerve. Trousseau's sign refers to carpopedal spasm and is seen in latent tetany

 

Rickets is a skeletal disorder that occurs due to vitamin D deficiency. Recall that vitamin D is necessary for the absorption of calcium( necessary for the formation of strong bones and teeth) and phosphorus. Signs of rickets include; weak and soft bones. Skeletal deformities, stunted growth etc.

45. After eating shrimps, a 25-year-old man suddenly developed skin itching, some areas of his skin became hyperemic or erupted into vesicles. Make the diagnosis:

Explanation

Urticaria (hives) is a vascular reaction of the skin marked by the transient appearance of smooth, slightly elevated papules or plaques (wheals) that are erythematous and that are often attended by severe pruritus. Individual lesions resolve without scarring in several hours. Most cases of urticaria are self-limited and of short duration; the eruption rarely lasts more than several days, but may be recurrent over weeks. Chronic urticaria is defined as urticaria with recurrent episodes lasting longer than 6 weeks. Mostly occurs as a reaction towards food, drugs, contact etc.

 

Urticaria pigmentosa is composed of persistent brown or red marks, made of collections of mast cells that swell and itch transiently when rubbed, similar to a hive.

46. A 35-year-old woman came to the family doctor with complaints of profuse menstruations that last up to 10 days. Gynecological examination shows that the uterine cervix is without changes, the uterus is in anterflexio, has normal size, is mobile and painless. The uterine appendages on the both sides are without peculiarities. The family doctor made the provisional\' diagnosis of abnormal uterine bleeding. What instrumental method of examination needs to be performed first to diagnose this pathology?

Explanation

Transvaginal ultrasound ( through the vagina ) is an ultrasound procedure used to examine the female reproductive organs. This includes the uterus, fallopian tubes, ovaries, cervix, and vagina.

 

Culdoscopy is used in examining the rectouterine pouch while colposcopy is restricted to examining the  cervix, vagina and vulva.

47. A 19-year-old young man was diving and hit his head on the bottom of the pool. He complains of pain in the neck, his head movements are limited and painful. During examination his head is bowed forward and to the right and the patient supports it with his hands. Palpation detects tense neck muscles and protruding spinous process of the IV cervical vertebra (C4). When pressure is applied to this process and to the head (axial load), the pain intensifies. Make the provisional diagnosis:

Explanation

From anamnesis, this patient had a trauma to the head which led to the results from palpation “ tense neck muscles and protruding spinous process of C4 vertebra. These findings help us put a provisional diagnosis of an Uncomplicated cervical fracture.
48. A patient is 45 years old. He was referred for a consultation with a psychiatrist due to complaints of abdominal pain and discomfort that occur in emotionally straining situations. Objectively no changes of the gastrointestinal tract were detected. The complaints emerged over 10 years ago against the background of severe alcohol poisoning. The patient has been repeatedly visiting gastroenterologists, who were unable to find any significant changes in the patient. The prescribed therapy was* ineffective. What is the likely conclusion?

Explanation

In Somatoform Autonomic dysfunction, the symptoms are presented by the patient as if they were due to a physical disorder of a system or organ that is largely or completely under autonomic innervation and control, i.e. the cardiovascular, gastrointestinal, respiratory, and  urogenital systems. Clinical and instrumental examination revealed no organic alterations in any system therefore indicating a somatoform autonomic dysfunction. 

 

Organic brain syndrome is defined as a state of diffuse cerebral dysfunction associated with a disturbance in consciousness, cognition, mood, affect, and behavior in the absence of drugs, infection, or a metabolic cause

49. A 21-year-old man was hospitalized on the 2nd day of the disease. His general condition is severe, body temperature is 39°(7, On his skin there are numerous irregular-shaped hemorrhagic elements. The diagnosis of meningococcemia was made. The next day his body temperature suddenly decreased, blood pressure was 80/40 mm Hg, pulse was 120/min. Acrocyanosis was detected. What complication did the patient develop?

Explanation

50. Forensic autopsy of the body of a 59-year-old man, who died suddenly at home without signs of violent death, shows pink skin and mucosa, liquid bright-red blood, and bright-red plethoric internal organs. Forensic toxicology testing detected 1.44°/00 of ethanol in the blood and carboxyhemoglobin levels of 55%. What is the cause of death?

Explanation

 

Notice that the carboxyhemoglobin level is 55% ( very high) - this indicates poisoning by carbon monoxide; the level of ethanol found in his blood is too little to cause poisoning or be the major reason behind his death. 

51. A 45-year-old man with a history of myocardial infarction developed a brief attack of palpitations, accompanied by the sensations of lack of air, fear, and vertigo. His blood pressure is 90/60 mm Hg. ECG during the attack shows extended QRS complex (0.13 seconds) with heart rate of 160/min., discordant shift of ST segment and T wave, dissociation of atrial and ventricular rhythm. What disturbance of cardiac rhythm is it?

Explanation

52. For 20 years the role of excessive weight in ischemic heart disease development among the working age male population over 40 was studied. It was determined that overweight men developed ischemic heart disease more often. What type of epidemiological study is it?

Explanation

 

In cohort study - both groups will have Ischemic Heart Disease, then you consider a risk factor that might have led to the Ischemic Heart Disease. In this question, you are studying overweight and normal weight in those with Ischemic Heart Disease and the study shows that overweight men developed ischemic heart disease more often.

In case control - you will compare those with Ischemic heart disease (Cases) and those without Ischemic Heart Disease (Control).

53. A 20-year-old woman complains of headaches, vertigo, tearfulness, vomiting, pain in the area of the heart, and tachycardia. The signs appear 6-7 days before menstruation and disappear in the first days of menstruation. Make the diagnosis:

Explanation

Premenstrual syndrome refers to a group of symptoms women experience a week or two weeks before menstruation. These symptoms may include; mood swings, abdominal pain , headaches, vomiting, vertigo, insomnia, changes in appetite and sex drive etc. Commonly prescribed medications for its management include; antidepressants, contraceptives, NSAIDs etc.

Polycystic ovarian syndrome is a disorder found in women of reproductive age commonly due to hormonal disorder (high level of androgens). This disorder is characterised by infrequent menstrual cycle, pelvic pain, weight gain etc. The ovaries mostly develop follicles (collection of fluids).

 

Algodysmennhorhea is characterised by painful menstruation or menstrual cramps.

54. A woman came to the gynecologist to plan her pregnancy She was advised to increase her intake of the products rich in folic acid, particularly soy beans, bread made of coarsely ground flour, fruits, leafy green vegetables. Such changes in her diet will work toward the prevention of:

Explanation

 

Folic acid is needed during pregnancy to prevent neural tube defects ( spina bifida) . An important role of folic acid is the formation of methionine from homocysteine using vitamin B12 as a cofactor. Adequate folic acid intakes can normalize high homocysteine levels via increased remethylation of homocysteine to methionine via 5-methyltetrahydrofolate-homocysteine methyltransferase (a.k.a.; methionine synthetase). Reduced folic acid intake is associated with hyperhomocysteinemia. Hyperhomocysteinemia is recognized as an independent risk factor for atherosclerosis of the coronary, cerebral, and peripheral vasculature. There is mounting evidence that elevated plasma homocysteine (and therefore decreased serum methionine) contributes to congenital neural tube defects.

55. A 48-year-old woman complains of disturbed menstrual cycle: her periods last for 7-9 days and are excessively profuse throughout the last half-year. She notes occasional hot flashes in her head, insomnia, irritability and headaches. Her skin is of normal color. Blood pressure _ 150/90 mm Hg, pulse - 90/min., rhythmic. The abdomen is soft and painless. Bimanual examination shows no uterine enlargement, the appendages cannot be detected. The vaginal fornices are free. What is the most likely diagnosis?

Explanation

Cimacteric syndrome is seen in women in Perimenopausal or postmenopausal age; they experience a decrease in ovarian function and decrease in female haormones. Climacteric syndrome is defined as gradual changes of ovarian function that start before menopause. Notable signs include; insomnia, dizziness, anxiousness and nervousness, poor sleep quality, hot flashes etc, 

Polycystic ovarian syndrome is a disorder found in women of reproductive age commonly due to hormonal disorder (high level of androgens). This disorder is characterised by infrequent menstrual cycle, pelvic pain, weight gain etc. The ovaries mostly develop follicles (collection of fluids). 

Premenstrual syndrome refers to a group of symptoms women experience a week or two weeks before menstruation. These symptoms may include; mood swings, abdominal pain , headaches, vomitting, vertigo, insomnia, changes in appetite and sex drive etc. 

 

Adrenogenital syndrome is also known as congenital adrenal hyperplasia.

56. A pregnant woman at 34 weeks of gestation underwent dopplerometry of umbilical circulation, which revealed a reverse diastolic component. Objectively the height of the uterus is 27 cm above the pubic bone, the head of the fetus is mobile and located above the entrance to the lesser pelvis. Fetal heartbeat is 136/min. Vaginal examination shows that the uterine cervix is closed, its length is 3 cm. What tactics should the obstetrician choose?

Explanation

In a normal situation, umbilical arterial flow should always be in the forward direction in both systole and diastole. 

Reversal of umbilical artery end-diastolic flow (REDF) or velocity is often an ominous finding if detected after 16 weeks. The feature is seen as a result of a significant increase in resistance to blood flow within the placenta and often represents a "tip of the iceberg" where there is a much larger underlying pathology.

However, during the first 16 weeks, a reversal in end-diastolic flow can be a normal finding due to the low resistance arcuate arteries and intervillous spaces not yet being formed.

Abnormal umbilical artery flow with absent or reversed end-diastolic velocity during pregnancy is a strong indication of placental insufficiency. Another supporting information here is the fact that there is an intrauterine growth restriction (IUGR). A pregnancy of 34weeks should have the Uterine height at 32-36cm. Anything less is often indicative of IUGR and a value above this range could be a case of macrosomia.

In general, absent and reversed end-diastolic flow of the fetal umbilical arteries are associated with poor neonatal outcomes, ranging from premature delivery and stillbirth to postnatal neurodevelopmental impairment

 

So, in this case, an Urgent Cesarean section can save both the fetus and the mother.

57. A 26-year-old woman was hospitalized into the gynecological department with complaints of body temperature up to 38.2°C, fever, general weakness, and dirty- red blood discharge from her genital tracts. She is hemodynamically stable. Two days ago she underwent a medical abortion on the 8th week of pregnancy Ultrasound detects the remains of the fertilized egg in her uterine cavity What are the tactics of the patient management in this case?

Explanation

A vacuum aspirator is used to induce abortion, evacuate incomplete miscarriage or retained pregnancy tissue and for endometrial biopsy. Notice that this patient underwent a medical abortion but the result from ultrasound detects the remains of the fertilised eggs in her uterine cavity - this is a direct indication to carry out vacuum aspiration.

 

Extirpation of the uterus also called total hysterectomy refers to the surgical removal of the entire uterus; it may also involve the cervix and ovaries.

58. A 23-year-old woman with type 1 diabetes mellitus during the 2nd week of community-acquired pneumonia developed 筹 nausea and vomiting. In the evening she has lost her consciousness and was hospitalized. Objectively the patient\'s skin is pale and dry Her respiration is loud, the tongue is dry with brown deposit. Her heart rate is 129/min., blood pressure is 85/50 mm Hg. Palpation of the patients abdomen provokes no response. The liver is +3 cm. Acetone test is markedly positive, blood glucose is 26 mmol/L. Make the provisional diagnosis:

Explanation

Notice that this patient presents with a type 1 diabetes ( insulin dependent). in the absence of insulin, ,the body tends to release more glucose from the liver and free fatty acids from adipose tissue; these free fatty acids are then broken down into ketone bodies (acetoacetate and β-hydroxybutyrate) through the process of beta oxidation. Accumulation of these ketone bodies in the blood leads to metabolic acidosis due to the fact that ketone bodies reduce the blood pH level. The breath of a person with Diabetic ketoacidosis is usually associated with a specific smell ‘Fruity’ or “acetone”. 

 

A diabetic hyperosmolar coma is caused by severe dehydration and very high blood glucose levels (hyperglycaemia). The kidneys respond to high levels of blood glucose by doing their best to remove it, along with a great deal of water. The person experiencing diabetic hyperosmolarity will be very thirsty coupled with a high level of osmolarity level from biochemical analysis. 

 

Lactic academic coma occurs as a result of a buildup of lactic acid in the blood.

 
59. On the 5th day after giving birth a postparturient woman complains of a pain in her left mammary gland and body temperature up to 38.1°(7. Examination shows that her mammary gland is enlarged and painful on palpation, the nipple is edematous and has fissures, the upper external quadrant of the gland is hyperemic. Name the measures that would have prevented the development of this complication in the patient:

Explanation

Mastitis is inflammation of the breast tissue and can be broken down into lactational and non-lactational mastitis. Lactational mastitis is the most common form of mastitis. Lactational mastitis, also known as puerperal mastitis, is typically due to prolonged engorgement of milk ducts, with infectious components from the entry of bacteria through skin breaks. Patients can develop a focal area of erythema, pain, and swelling, and can have associated systemic symptoms, including fever. This occurs most commonly in the first six weeks of breastfeeding but can occur at any time during lactation, with most cases falling off after 3 months. Lactational mastitis is most commonly caused by bacteria that colonize the skin, with Staphylococcus aureus being the most common, in this case an antibiotic may be administered to fight the infection.

The first line of treatment is self-help remedies, such as ensuring that the breast is drained properly during feeds by breastfeeding regularly or expressing the milk. And after a feed, gently express any leftover milk.

 

Every newborn should be fed on demand. No schedule should be followed. Whenever the baby wants the milk, breastfeeding should be allowed. Breastfeeding should not be stopped because of cracks or pain in the breast.

60. On ultrasound of the thyroid gland, a 47-year-old woman presents with a hypoechoic node 1.6 cm in diameter with blurred margins and intranodular hypervascularization. The doctor suspects thyroid carcinoma. What method should be used to verify the diagnosis?

Explanation

A thyroid fine needle aspiration biopsy is a procedure that removes a small sample of tissue from your thyroid gland. Cells are removed through a small, hollow needle. The sample is sent to the lab for analysis. In some cases, hard nodules form inside the gland. Most times, the nodules are not dangerous. But in some cases they can be thyroid cancer. A thyroid fine needle aspiration biopsy can take a sample from the nodule to test for cancer.

 

Thyroid scan (thyroid scintigraphy) is a nuclear medicine examination used to evaluate thyroid tissue. Clinical indications include; functional status of a thyroid nodule, thyrotoxicosis: differential diagnosis and thyroid cancer. Note that an ultrasound procedure has already been carried out therefore, carrying out a thyroid scintigraphy will just be a repetitive test.

61. In a pediatric clinic, located in a rural area, there are 9 children, who simultaneously fell ill. The following signs were detected: low physical activity, acrocyanosis of the nasolabial triangle and fingertips, mucosal cyanosis, tachycardia, dyspnea. It was determined that all the sick children were fed with a formula that was dissolved in the water taken from a dug well. Laboratory analysis revealed high levels of methemoglobin in the blood of the children. These signs can be caused by increased content of a certain element in the water. Name this element:

Explanation

Methemoglobinemia is an unusual and potentially fatal condition in which hemoglobin is oxidized to methemoglobin and loses its ability to bind and transport oxygen. The most common cause of methemoglobinemia is the ingestion or inhalation of oxidizing agents such as nitrates or nitrites. Signs include dizziness, headaches, cyanosis, dyspnea , tachycardia etc. Treatment of this condition involves the use of methylene blue and vitamin C; other substances that can cause this state (methemoglobinemia) include dapsone and benzocaine. 

 

Lead poisoning is characterised by lead lines on the gingiva (burton lines), encephalopathy, erythrocyte basophilic stippling, abdominal colic, sideroblastic anemia and wrist or foot drop. Dimercaprol ( chelating agent) is used in treatment.

62. A 20-year-old man suffers from headache,。general weakness, and face edema that appears in the morning. 18 days earlier he had a case of tonsillitis. Objectively his skin is pale, there are edema under his eyes. Heart rate is 60/min., blood pressure is 185/100 mm Hg, The sign of costovertebral angle tenderness (punch sign in the lumbar region) is negative. Urinalysis: color of ’’meat slops? protein - 4.5 g/L, altered erythrocytes - 40-45 in the vision field, leukocytes - 5-6 in the vision field. 24- hour diuresis is 400 mL. What is the most likely diagnosis?

Explanation

Note that 18 days prior, this patient had a case of tonsillitis; this raises the idea of a group A beta hemolytic streptococci infection. This infection is most likely to cause glomerulonephritis (post streptococcal glomerulonephritis). Poststreptococcal (or post infectious) glomerulonephritis (PSGN) refers to acute glomerular inflammation that results from a preceding infection with nephritogenic strains of streptococci. Deposition of immune complexes containing the streptococcal antigen within the glomerular basement membrane results in complement activation and subsequent damage to the glomeruli. PSGN typically presents as a nephritic syndrome with hematuria, mild proteinuria, edema, and hypertension. Elevated antistreptolysin O titers (ASO), low complement levels, and elevated creatinine support the diagnosis.
63. An 18-year-old patient always obeys others and adapts his needs to the demands of the people on whom he depends. He excessively defers to their wishes and makes them responsible for his wellbeing, cannot defend his interests and needs support from other people. Such psychic profile has been formed in the childhood, remains unchanged, and hinders adaptation. What psychic disorder is observed, in this patient?

Explanation

People with dependent personality disorder may submit to the will of others in a misguided attempt to extract a promise of care and protection. They may think of themselves, or present themselves, as unable to cope with everyday life on their own. At the same time, they may fear that a show of confidence or competence will lead to rejection and abandonment. They demand advice and reassurance when making even minor decisions. They take no initiative and let others assume responsibility for their lives.

 

Anankastic personality disorder is also  known as Obsessive compulsive disorder (OCD),a disorder characterized by preoccupation with orderliness, perfectionism, and mental and interpersonal control, at the expense of flexibility, openness, and efficiency.

64. A 35-year-old woman complains of a pain in her right axillary region. She has been suffering from this condition for a week. Her body temperature is 38°C. In the right axillary region there are 2 formations, 2r cm in size each. The skin over the formations is dark red and thin. Palpation produces a yellow-white discharge from the fistular openings. What is the most likely diagnosis?

Explanation

Carbuncle: red, swollen, painful clusters of boil that are connected to each other under the skin. 

Furuncle: refers to a Boil, forms around the hair follicle and contains pus.

 

Hydradenitis: ‘acne inversa’ , inflamed and swollen lumps found in the axillary region ( armpit), under the breast, groin. These lumps most times break open forming a fistula or tunnels

65. During a regular examination, an 8- year-old girl with type I diabetes mellitus presents with a swelling on the anterior surface of her hip. The swelling is 3 cm in diameter; dense, painless on palpation. The skin over this formation has normal color and temperature. Localization of the swelling matches the place where the girl usually receives her insulin injections. What is the most likely cause of this clinical presentation?

Explanation

Lipohypertrophy Insulin Injection

Lipoatrophy Insulin Injection

The background information here is that - this young lady with Type I Diabetes Mellitus presents with a 3cm swelling on the anterior surface of the hip. 

Abnormal reaction in subcutaneous fat to insulin is called lipodystrophy which can be either hypertrophic or atrophic. Lipodystrophy is an exclusive complication of lean children and young Type 1 diabetic, although rarely can be seen in Type 2 diabetic.

Insulin lipohypertrophy denotes a benign tumor like swelling of fatty tissue at the injection site secondary to lipogenic effect of insulin, whereas lipoatrophy is considered an adverse immunological side effect of insulin therapy. Since the advent of recombinant human insulin and analogue, lipoatrophy, has virtually disappeared, whereas, lipohypertrophy still remains a serious local problem of insulin therapy.

With a swelling of 3cm, the best answer choice is development of hypertrophic lipodystrophy. Atrophic lipodystrophy will not have associated swelling, instead a depression should be observed.

 

Prevention, to avoid lipodystrophy, should take first place as there is little cure to it, and the best way is to educate the patient about rotating injection sites.

66. A 52-year-old woman has been suffering for 2 years from dull, occasionally exacerbating pain in her right subcostal area, occurring after eating high-fat foods, bitter taste in her mouth in the morning, constipations, and flatulence. Objectively she is obese, her body temperature is 36.9°C; there is a coating on the root of her tongue; the abdomen is moderately distended and painful in the area of gallbladder projection. What examination would be the most helpful for diagnosis making? ,

Explanation

 

Notice that this patient presents with signs in relation to gallbladder or liver pathology ( bile in general). In order to get the correct diagnosis, an Ultrasound of the liver, gall bladder and surrounding abdominal organs should be carried out.

67. A family doctor performed an external obsterical examination of a pregnant woman and determined that her uterine fundus is located at the level of the navel. What is the most likely term of pregnancy in this woman?

Explanation

Location of Uterine Fundus in relation to term of Pregnancy. 

Pubic Symphysis - 12-15 weeks 

Umbilicus             - 20-24 weeks

Xiphoid process of sternum- 36-40 weeks

 

Note that after 36 weeks of pregnancy, the fundus length is about 32- 36cm.

68. What should be prescribed as secondary prevention drugs for a patient with atrial fibrillation after an ischemic stroke caused by cardiac embolism?

Explanation

Dabigatran, rivaroxaban, apixaban, and edoxaban (direct acting oral anticoagulants) are approved for the lowering the risk of stroke and embolism in NVAF ( non-valvular atrial fibrillation) as well as deep vein thrombosis and pulmonary embolism treatment/prophylaxis. Unique indications include betrixaban for prophylaxis of venous thromboembolism (VTE) in hospitalized patients for an acute medical illness, and rivaroxaban in combination with aspirin to reduce major cardiovascular events in patients with chronic coronary artery disease (CAD) or peripheral artery disease.
69. A 55-year-old woman complains of thyroid gland enlargement that can be observed throughouth the last 2 years and a discomfort during swallowing. Objectively she has signs of hypothyroidism. The thyroid gland on palpation is dense, nonfused with the surrounding tissues and mobile on swallowing. The regional lymph nodes are not enlarged. In the serum there are antithyroid antibodies detected. What is the most likely diagnosis?

Explanation

 

The presence of antithyroid antibodies helps us understand that there is an autoimmune attack on the thyroids- This refers to Hashimoto’s thyroiditis; the most common cause of hypothyroidism. It is characterised by lymphocyte infiltration of the stroma of the thyroid gland leading to its destruction and signs of hypothyroidism. Other antibodies that can be found include those against thyroid peroxidase and TSH receptors. For its treatment, a replacement therapy is carried out using L-thyroxine.

70. A 12-year-old girl after a case of respiratory infection developed dyspnea at rest, paleness of skin. Heart rate is 110/min., BP is 90/55 mm Hg. Heart sounds are muffled. Borders of relative heart dullness: right - the parasternal line, upper - the III rib, left - 1,0 cm outwards from the midclavicular line. Make the provisional diagnosis:

Explanation

Notice that all these heart changes took place in the background of a ‘respiratory infection’. Viral infections are a leading cause of myocarditis. Inflammation occurs during the course of infection, putting stress on the heart that remains even after the infection is resolved. Cancer, bacterial infections and other contagious diseases can also cause myocarditis.

 

In hypertrophic cardiomyopathy, the heart muscles become abnormally thick ( hypertrophied); this leads to issues relating to the pumping ability of the heart.

71. A 19-year-old young man complains of cough with expectoration of purulent sputum in amount of 100 mL per day, hemoptysis, dyspnea, increased body temperature up to 37.8°C, general weakness, weight loss. The patient\'s condition lasts for 4 years. Exacerbations occur 2-3 times a year. The patient presents with malnutrition, pale skin, cyanosis of the lips, drumstick (clubbed) fingers. Tympanic percussion sound in the lungs, weakened respiration, and various numerous moist crackles in the lower pulmonary segments on the left can be observed in this patient. In blood: erythrocytes - 3.2 • 1012 /L leukocytes -8.4 • 109 /L,ESR- 56 mm/hour. On X- ray: lung fields are emphysematous, the left pulmonary root is deformed and dilated. What is the most likely diagnosis?

Explanation

Bronchiectasis is a condition in which damage to the airways causes them to widen and become flabby and scarred (X-ray findings - left lung root is deformed and dilated). The airways are tubes that carry air in and out of your lungs.

Bronchiectasis usually is the result of an infection or other condition that injures the walls of your airways or prevents the airways from clearing mucus.

The initial airway damage that leads to bronchiectasis often begins in childhood. However, signs and symptoms may not appear until months or even years after you start having repeated lung infections.

The most common signs and symptoms of bronchiectasis are:

  • A daily cough that occurs over months or years.

  • Daily production of large amounts of sputum (spit). Sputum, which you cough up and spit out, may contain mucus (a slimy substance), trapped particles, and pus.

  • Shortness of breath and wheezing (a whistling sound when you breathe).

  • Chest pain.

  • Clubbing (the flesh under your fingernails and toenails gets thicker).

 

Over time, it may progress to more serious symptoms like coughing up blood (hemoptysis) or bloody mucus and feel very tired. Children may lose weight or not grow at a normal rate.

72. A patient 1 year ago had a Q wave myocardial infarction of the posterior wall of the left ventricle. For the last 2 weeks he has been suffering from daily attacks of atrial fibrillation and bradycardia episodes, accompanied by bouts of vertigo. What tactics is the most advisable in this case?

Explanation

 

This patient is experiencing a sinus node dysfunction in combination with a tachyarrhythmia; this condition is referred to as a tachy-brady syndrome which is a major cause of intermittent bradycardia ( an indication for Pacemaker replacement). Sinus node dysfunction encompasses other pathologies which include; SA block, sinus arrest and sinus inhibition, sinus bradycardia etc. Other conditions that require a Pacemaker replacement include; High grade AV block, Hypersensitive carotid sinus syndrome and neurocardiogenic syncope, congenital heart disease etc.

73. A 58-year-old woman has type 2 diabetes mellitus that is compensated with diet and metformin. She prepares for cholecystectomy. Objectively: her height is 164 cm, weight is 90 kg, heart rate is 72/min., blood pressure is 130/80 mm Hg. Her abdomen is soft, painful in the right subcostal region. The liver is not enlarged. Fasting glucose - 6.2 mmol/L. Glycated hemoglobin - 6.5%. What further tactics of sugar-lowering therapy should be chosen in this case?

Explanation

Notice that this patient’s blood glucose and glycated hemoglobin levels are marginally high; she has also been on a diet regime coupled with metformin intake. On the basis that she is being prepared for a cholecystectomy ( a surgical procedure), she needs to be administered a short acting insulin  ( short acting just to last the period of the surgical process).
74. A 23-year-old woman came to the gynecological clinic. She complains of pain, itching, and burning in her vulva, general weakness, indisposition, elevated body temperature up to 31.2°C, and headache. On examination in the vulva there are multiple vesicles up to 2-3 mm in diameter with clear contents against the background of hyperemia and mucosal edema. Make the provisional diagnosis:

Explanation

Genital herpes is an infection caused by Herpes simplex 2 virus. It is characterised by a red, swollen, pruritic genital accompanied by an unusual discharge. Other findings include the presence of single or disseminated vesicular lesions; these lesions are usually found around the genitals and anus and are accompanied by an itchy and burning sensation.  

 

Primary syphilis is characterised by the presence of a hard, painless chancre ( a painless genital ulcer).

75. A 5-year-old child had acute onset of the disease that manifested in body temperature up to 39.5°C, marked chills,, weakness, inertness, skin pallor, and headache. 8 hours later a hemorrhagic rash developed on the skin of the buttocks and legs. The child is sluggish, the body temperature has dropped, blood pressure is 80/40 mm Hg, respirations are 28-30/min., diuresis is decreased. Make the provisional diagnosis:

Explanation

Meningococcemia is defined as dissemination/spread of meningococci (Neisseria meningitidis) into the bloodstream. Patients with acute meningococcemia may present with (1) meningitis (2) meningitis with meningococcemia, or (3) meningococcemia without clinically apparent meningitis. This patient appears to have the third presentation, Clinical presentation include; petechial/hemorrhagic skin rash usually located on the trunk and legs and may rapidly evolve into purpura,  malaise, weakness, myalgias, headache, nausea, vomiting, and arthralgias. 

Henoch-schonlein purpura ( hemorrhagic/ IgA vasculitis) is a condition in which small blood vessels are inflamed. It is an acute immune complex-mediated small vessel vasculitis that most commonly occurs in children. typically presents with a tetrad of symptoms: palpable purpura, arthritis/arthralgia, abdominal pain, and renal disease; although not all patients show the complete four symptoms. 

 

Thrombotic thrombocytopenic purpura (TTP) is a thrombotic microangiopathy, a condition in which microthrombi, consisting primarily of platelets, form and occlude the microvasculature (i.e., the arterioles and capillaries).

76. A 75-year-old man in a severe condition suffers from dyspnea at rest, marked weakness, and arrhythmia. Abdominal aortic pulsation is observed, further on there is a systolic murmur detected. Palpation reveals a volumetric formation in the mesogastrium. Blood pressure is 70/40 mm Hg. There is no pulsation over the femoral arteries. Oliguria is detected. Which diagnosis is the correct one?

Explanation

An aortic aneurysm is a localised pathologic dilation of the aorta; may cause abdominal and/or back pain which is a sign of  leaking, dissection or imminent rupture. This patient is most likely having an aneurysm of the abdominal aorta which usually presents as a palpable abdominal pulsatile mass - Notice that in this patient, abdominal pulsation is observed coupled with a volumetric formation in the mesogastrium.

Acute cardiac aneurysm or a Cardiosclerotic aneurysm are common complications of a recent Myocardial Infarction. It refers to ventricular aneurysms which leads to decreased contractility of the heart and reduction of the flow of oxygen-rich blood to the body, causing heart failure and death. A cardiac aneurysm will rather show retrosternal symptoms instead of a mesogastric one as seen in this case.
77. When planning treatment of a patient, it was decided to use a medicine with evidence level A. What trials produce the evidence that allows to classify the medicine as level A?

Explanation

As the name suggests, evidence-based medicine (EBM), is about finding evidence and using that evidence to make clinical decisions. A cornerstone of EBM is the hierarchical system of classifying evidence. This hierarchy is known as the levels of evidence.

Randomized Clinical or Controlled Trials: Include a randomized group of patients in an experimental group and a control group. These groups are followed up for the variables/outcomes of interest. Randomized controlled trials (RCTs) are considered the gold standard in modern medicine for determining the efficacy of a treatment. Individual RCTs are level 1b evidence. Systematic reviews of homogenous RCTs are regarded as the highest level of evidence—level 1a. These systematic reviews consist of information synthesized from individual, well-designed RCTs where participants are similar and have equal chances of being assigned to an intervention group, a control group, or a placebo group.

Grade of

Recommendation

Level of

Evidence

Type of Study

A

1a   

Systematic review of (homogeneous) randomized

controlled trials

A

1b

Individual randomized controlled trials (with narrow

confidence intervals)

B

2a

Systematic review of (homogeneous) cohort studies

of "exposed" and "unexposed" subjects

B

2b

Individual cohort study / low-quality randomized

control studies

B

3a

Systematic review of (homogeneous) case-control studies

B

3b

Individual case-control studies

C

4

Case series, low-quality cohort or case-control studies

D   

5

Expert opinions based on non-systematic reviews of

results or mechanistic studies

 

Two things to remember:

1. Studies in which randomization occurs represent a higher level of evidence than those in which subject selection is not random.

2. Controlled studies carry a higher level of evidence than those in which control groups are not used.

 

Expert Consensus - Level D

 

Case Control studies - Level B

 
78. After the pregnant woman’s waters broke, it was noted that they are significantly contaminated with meconium. Upon birth, the baby is not breathing, remains inert, the skin is cyanotic and covered in meconium, heart rate is 98/min. What resuscitation measures should be taken after the baby is born?

Explanation

On the basis of evidence, it has been recommended that all babies born through thick meconium should have their tracheas intubated so that suctioning of their airways can be performed. The aim is to reduce the incidence and severity of meconium aspiration syndrome. 

 

Meconium is the first stool of a newborn baby, greenish‐black in colour and rather sticky in consistency.  If a baby is distressed before birth meconium may be passed into the amniotic fluid and if the baby then gasps the meconium‐stained fluid may pass into the lungs.  Once in the lungs the meconium can make the baby ill by obstructing the airways and causing inflammation this is called meconium aspiration syndrome.  It is possible to reduce the amount of meconium getting into the lungs by sucking it from the baby’s throat and windpipe (trachea) soon after birth.

79. The 5-year-old child has been ill for 2 weeks. Cough attacks developed first and were then followed by reprises. During coughing the child’s face turns red and cervical veins bulge. The cough attacks induce vomiting. X-ray shows intensified bronchial pattern. Blood test: leukocytes - 16.109 /L , lymphocytes - 72%, erythrocyte sedimentation rate - 4 mm/hour. What is the most likely diagnosis?

Explanation

 

Pertussis, or whooping cough, is a highly infectious disease of the respiratory tract caused by the gram-negative bacteria Bordetella pertussis. The disease is mainly transmitted via airborne droplets and most commonly occurs in children. Typically, pertussis manifests in three stages, with the second and third stage characterized by intense paroxysmal coughing that is followed by a distinctive whooping sound on inhalation (reprise) and, in some cases, vomiting.

80. A 48-year-old woman was delivered into the surgical unit with wounds in her thigh. On examination the wound surface has a dirty-gray coating with unpleasant sweet smell. The wound content resembles a raspberry jelly. Skin tissues around the wound are glossy and turgid. palpation reveals moderate  crepitation in the tissues. What bacteria is most likely to cause such inflammation?

Explanation

From the description; dirty gray coating with unpleasant sweet smell, resembling raspberry jelly with moderate criterion on palpation - this fits the description of a Gangrenous infection (gangrene); an infectious process caused by Clostridium Perfringens. Cl. Perfringens is a gram positive, rod shaped, spore forming anaerobic bacteria which is a major cause of food poisoning and gas gangrene.

 

Blue pus bacillus refers to pseudomonas aeruginosa

81. A patient underwent suture plication of the perforated duodenal ulcer. On the 3rd day after the operation he started producing a large amount of discharge from the abdominal drain tube. The discharge contains bile and has high amylase levels. What complication occurred in the patient?

Explanation

At the end of the operation (suture plication), a drainage tube is inserted into the abdomen for two purposes: firstly, the detection of any fluid collections within the abdomen (intra‐abdominal collections), usually resulting from intestinal secretions; and secondly, as the treatment of intra‐abdominal collections, so that fluid collection decreases or, at least, does not worsen within the abdomen. The fluids from the drain can be tested for enzymes like amylase (one of the contents of the pancreatic juice which digests carbohydrates), bile secretions or intestinal juice to find out whether the fluid in the drain is because of a suture leak. A case of suture leaks often presents as a sequelae to suture incompetence.

 

If there is a high suspicion of a pancreatic/bile leak (as seen in this patient), further scans are performed to confirm it or to rule it out. If the leak is major and the patient is unwell, urgent reoperation may be required.

82. A 40-year-old victim of a traffic accident sustained the following injuries: closed diaphyseal femur fracture, brain concussion, multiple rib fractures, hemopneumothorax, degloving skin injuries. What injuries require the most urgent attention?

Explanation

In resuscitating a patient, C-A-B sequence is very important. 

C is to help maintain blood circulation; A - airway and B - Breathing.

Hemopneumothorax can be considered under these 3.

Hemo - active internal hemorrhage into the pleural cavity.

Pneumo - ongoing air leakage into the pleural cavity (rib fractures).

If these are not attended to first, eventually, the lungs will not be able to expand, hence, no gaseous exchange compromising both A- airway and B- breathing. If no action is taken, and the hemorrhage and air leakage into the pleural cavity continues, it will eventually lead to hypovolemia and the accumulated blood will compress the mediastinum which houses the heart thereby preventing the heart from contracting as well and eventually compromises C- circulation.  Once this is attended to, the other injuries can receive proper care as well but hemopneumothorax comes first.

Now, considering this patient, there is no active external bleeding going on. Closed diaphyseal femur fracture just needs immobilization as first aid. It is not open and the question did not state that there is an injury to a major blood vessel. In Degloving shin injuries, if there is an active bleeding, the vessel can be ligated and the wound dressed.

 

The most common and least serious type of traumatic brain injury is called a concussion. A concussion is most often caused by a sudden direct blow or bump to the head.

83. A 20-year-old woman, gravida 2, para 1 has been in labor for 4 hours. Her condition is satisfactory Moderately painful contractions occur every 3 minutes and last for 35-40 seconds. The waters have not burst yet. The fetus is in longitudinal position. Fetal heartbeats are 136/min., clear and rhythmic. Major segment of the fetal head is engaged to the pelvic inlet. Vaginal examination shows smooth cervix of 6 cm, amniotic sac is intact, sagittal suture is in the left oblique diameter, occipital fontanel is on the right near the symphysis pubis. What stage of the labor is it?

Explanation

The first stage of labor is considered to last from the onset of regular uterine contractions to full dilation of the cervix. divided into three phases: a latent phase, an active phase, and a transition phase. During the latent phase there is more progress in effacement (stretching and thinning) of the cervix and little increase in descent. During the active phase and the transition phase there is more rapid dilation of the cervix and increased rate of descent of the presenting part.

The second stage of labor lasts from the time the cervix is fully dilated to the birth of the fetus. It takes an average of 20 minutes for a multiparous woman and 50 minutes for a nulliparous woman.

The third stage of labor lasts from the birth of the fetus until the placenta is delivered.

 

The presented cervix is 6cm ( dilated to an extent), Major segment of the fetal head is engaged to the pelvic inlet - indicating either an active or transition phase.

84. A 22-day-old infant developed subcutaneous red nodes from 1.0 to 1.5 cm in size on the scalp; later the nodes suppurated. lemperature increased up to 377°C, intoxication symptoms appeared, regional lymph nodes enlarged. Complete blood count: anemia, leukocytosis, neutrocytosis, raised ESR. What diagnosis will you make?

Explanation

pseudofurunculosis is observed in neonates and infants. The disease begins with the appearance of superficial pustules in the ostium of sweat glands (periporitis). Caused by staphylococcus aureus, which penetrates in the depth of the sweat gland and causes the forming of deep indurated painful nodules. The nodules are very similar to furuncles, but they have not got core in the center (hence the name pseudofurunculosis).

 

Vesiculopustulosis is a widespread purulent disease, which appears in the first years of life. In the ostium of the sweat glands numerous pustules appear, filled with white yellow matter, the size of a pin head to a small pea, they do not merge with each other and are surrounded by bright edematous circles.

85. A 26-year-old man complains of chills, rhinitis, dry cough, and fever up to 38°C. Examination shows him to be in a moderately severe condition; there are small pale pink non-merging spots on the skin of his back, abdomen, and extremities. Palpation reveals enlarged occipital and axillary lymph nodes. No information about vaccination history could be obtained. What is the likely etiology of this disease?

Explanation

Rubella ( accused by rubella virus) is an acute viral infection characterised by a short prodromal period, Exanthema ( rash) during three days and lymphadenopathy. The type of rash found is roseola and small macula-papula. Note that the rash elements do not merge ( non-merging spots) and are usually concentrated on the extensor surface of extremities, back, buttocks and outer surface of thigh.

Epstein-barr virus is  one of the causative agents of Infectious mononucleosis- a disease characterised by prolonged fever, systemic lymphadenopathy, acute tonsillitis, acute adenoiditis, hepatosplenomegaly, along with typical blood changes  such as lymphocytosis, monocytosis, presence of specific cells called atypical mononuclear cells or virocytes.

 

Mumps infection is a systemic disease characterised by hyperthermic syndrome, symmetrical or unilateral swelling of parotid glands, epididymo-orchitis etc

86. A 38-year-old patient has been delivered by an ambulance to a surgical department with complaints of general weakness, indisposition, black stool. On examination the patient is pale, there are dotted hemorrhages on the skin of his torso and extremities. On digital investigation there are black feces on the glove. Blood test: Hb-108 g/L, thrombocytopenia. Anamnesis states that similar condition was observed 1 year ago. Make the diagnosis:

Explanation

Immune thrombocytopenic purpura (ITP) is a clinical syndrome in which a decreased number of circulating platelets (thrombocytopenia) present as a bleeding tendency, easy bruising (purpura), or extravasation of blood from capillaries into skin and mucous membranes (petechiae). Recall that the patient presents with dotted hemorrhages on the skin (petechiae) and on analysis, thrombocytopenia. Caused by the binding of an autoantibody (specifically IgG) to platelets leading to platelet destruction.

In Hemophilia, patients will present with bleeding from joints (hemarthroses), easy bruising or a case of bleeding after surgery, dental procedure. Has three forms; Hemophilia A ( Factor VIII deficient), Hemophilia B ( factor IX deficient), C ( factor XI def.).
87. The body of a citizen was found at the place of his dwelling. On his face, neck, and hands there were detected irregular-shaped wounds, varying from 2x3 cm to 4x5 cm in size. The skin and underlying tissues are absent in the wounds. The margins of the wounds are uneven, with major and minor scalloping along the edges and no signs of bleeding. What is the initiating mechanism of these wounds?

Explanation

Domestic animals, like dogs and cats, are responsible for most animal bites. Animal bites can range from small injuries that barely break the skin to severe wounds that can cause a person to lose the use of a hand, eye, or foot or even bleed to death. Animal bites typically cause irregular-shaped, uneven wound edges on the skin, as well as lacerations (jagged, uneven cuts) and abrasions (scrapes or grazes).
88. A multigravida, labor II, 36-37 weeks of gestation, has gone into labor Her waters broke 8 hours ago, the labor activity continues for the last 4 hours, it is regular, with contractions that last 35 seconds and occur every 3-4. minutes. The child is in the cephalic presentation, with the head pressed to the entrance into the lesser pelvis. The parturient woman complains of a sudden sharp abdominal pain. Her pulse is 100/min., blood pressure is 110/70 - 100/70 mm Hg. The uterus is tense and does not relax between the contractions. Fetal heartbeat is muffled - 100/min. The amniotic fluid is blood-colored and continues to leak. What is the most likely diagnosis?

Explanation

Placenta abruptio (also called placental abruption) is when the placenta separates from the inner wall of the uterus before the baby is born.  Common symptoms include sudden-onset abdominal pain, contractions that seem continuous and do not stop, vaginal bleeding, enlarged uterus disproportionate to the gestational age of the fetus, decreased fetal movement, and decreased fetal heart rate. Notice that this patient complains of a sudden sharp abdominal pain, The uterus is tense and does not relax between the contractions, blood coloured amniotic fluid- indicating uterine bleeding; all these signs help indicate the Premature detachment of the normally positioned placenta ( Placenta abruptio). 

 

In Placenta Previa, the placenta abnormally covers the cervix ( the uterine outlet); it is usually accompanied by a painless bright red vaginal bleeding.

89. A 45-year-old woman complains of increasing body weight throughout the last year. Examination revealed moon face syndrome, brittle hair; hirsutism, stretch marks on the abdomen, and disproportionally thin limbs. The patients height is 162 cm, her body weight is 94 kg, her body mass index is 35.8 kg/m2 . What type of obesity is it?

Explanation

90. A 2.5-year-old child is ill for the second day The onset of the disease was associated with the temperature up to 378°C, a single bout of vomiting, and watery diarrhea up to 5 times per day During the second day vomiting occurred twice, body temperature is 38.0°C, the child has low appetite, watery diarrhea continues. .The treatment of the child should start with the following:

Explanation

Notice that this patient has lost lots of fluid; it was stated that the child experienced a single bout of vomiting and diarrhea 5 times on the first day followed by vomiting and continuous diarrhea on the second day. For the treatment of the child, we should start with oral rehydration therapy which involves the oral administration of  water which contains a certain amount of glucose and salts. Loperamide can later be used to control/stop diarrhea while an antibiotic regimen can be used if the causative agent was found to be of bacterial origin.
91. On the 3rd day after the artificial abortion the woman was hospitalized into the gynecological department in a severe condition with signs of intoxication, abdominal pain, and purulent discharge from the vagina. Objectively the patients condition is severe, her body temperature is 38.8°C, pulse is 100/min., blood pressure is 110/70 mm Hg? the uterus is soft, the uterine fundus is located at the level of the navel, there are positive signs of peritoneal irritation. What is the most likely diagnosis?

Explanation

The key finding here is the positive signs of peritoneal irritation which is usually seen in patients with peritonitis. Notice that this woman underwent an abortion 3 days ago; her current situation is a complication of the procedure carried out. Purulent discharge from the vagina indicates the presence of an infectious process. Pelviperitonitis refers to a pathology that involves the inflammation of the peritoneum and organs of the pelvis.

Metroendometritis – the inflammation of a muscular and mucous membrane of a uterine wall combining signs of a myometritis and an endometritis. The clinic of a sharp metroendometritis is defined by a hyperthermia, intoxication, pain in the bottom of a stomach, purulent bleach.

 

Salpingo oophoritis involves the inflammation of the uterine tubes and the ovaries.

92. You witnessed a car accident. When examining the place of the accident you noticed a man of about 30 years, who was hit by the car. He is unconscious. On his neck on the left there is a profuse hemorrhage with bright-red blood. How to stop this hemorrhage? 1

Explanation

For the temporary stop of hemorrhage; - the imposition of a pressure

bandage;

- the elevated position of the limb;

- digital pressure of the artery

throughout;

- maximum limb flexion in the joint;

- stop bleeding with a tourniquet;

- stop bleeding with a tourniquet;

- stop bleeding from the carotid artery according to the method of Mikulich. This patient is bleeding from the carotid artery ( bright red blood from the neck) therefore, the highlighted procedures should be followed.

93. A 27-year-old man complains of pain in his leg joints, purulent discharge from the eyes, and painful burning sensations during urination. The disease onset was acute. He has a history of influenza. The patient smokes and drinks alcohol in excess. In his line of work he is often away on business trips. What is the most likely etiological factor of this disease?

Explanation

 

This patient presents with Classical signs of Reactive arthritis ( He can’t See , Pee or Bend The Knee) ie, purulent discharge from the eyes, burning sensation while urinating and pain in his leg joints. Reactive arthritis is an autoimmune condition that develops in response to an infection in another part of the body. Coming into contact with bacteria and developing an infection can trigger reactive arthritis. Chlamydia trachomatis is the most common bacteria that triggers Reactive arthritis.

94. A 57-year-old man, a miner, complains of a pain in his chest, dyspnea on physical exertion, excessive sweating, constant subfebrile tempeature, and cough that produces blood-streaked sputum. He has been smoking for approximately 40 years (2 packs a day) and frequently has ’’pneumonias? Survey chest X-ray shows a triangular shadow in the middle lobe of the right lung. One of the apices of the shadow points to the lung root. Cardiac and mediastinal shadows are displaced toward the affected area. Make the provisional diagnosis:

Explanation

95. A district doctor has diagnosed one of his patients with dysentery. What accounting document reflects this type of morbidity?

Explanation

Dysentery is an intestinal infection that causes severe diarrhea with blood. In some cases, mucus may be found in the stool. Dysentery is usually spread as a result of poor hygiene. For example, if someone who has dysentery doesn’t wash their hands after using the toilet, anything they touch is at risk.

The infection is also spread through contact with food or water that has been contaminated with fecal matter. Careful hand washing and proper sanitation can help prevent dysentery and keep it from spreading.

Shigellosis and amebic dysentery are very common examples. It spreads rapidly in environments where people who don’t have dysentery come into contact with fecal matter from people who do have dysentery.

This contact may be through: contaminated food; contaminated water and other drinks; poor hand washing by infected people; swimming in contaminated water, such as lakes or pools and physical contact.

Children are most at risk of shigellosis, but anyone can get it at any age. It’s easily spread through person-to-person contact and by contaminated food and drink.

 

So, it should be reported as urgent to prevent further spread and limit the number of people that will be infected so as to locate the source of infection and those infected can be isolated.

96. A 31-year-old drug-addicted person complains of a cough with bloody expectorations, dyspnea, persistent fever, and leg edemas. The jugular veins are distended. There is a coarse pansystolic murmur detected above the base of the xiphoid process and in the second intercostal space on the left, close to the edge of the sternum. Heart sounds are clear; arrhythmia is detected, heart rate is 128/min” pulse - 82/min., blood pressure is 100/70 mm Hg. What is the most likely diagnosis?

Explanation

Infective endocarditis (IE) is an infectious inflammation of the endocardium that affects the heart valves. The condition is a result of bacteremia, which is most commonly caused by dental procedures, surgery, distant primary infections, and nonsterile injections. From anamnesis, we understand that this patient is a drug addict and most likely uses non sterile injections. Clinical features include constitutional symptoms (fatigue, fever/chills, malaise) in combination with signs of pathological cardiac changes (e.g., new or changed heart murmur, heart failure signs). Notice that this patient has a pansystolic (holosystolic) murmur (usually heard in tricuspid valve regurgitation which can be one of the cardiac changes in IE). 

 

Lutembacher syndrome is a very rare disorder that involves a congenital atrial septal defect coupled with an acquired mitral stenosis.

97. A 7-year-old boy after a fall from a height presents with rapid and shallow breathing and cyanotic face. The right half of his thorax is distended and takes no part in the respiration. Percussion detects tympanitis in the affected area, while auscultation detects no breathing there. What pathology is the most likely cause of this clinical presentation? What instrumental examination would be the most informative in this case?

Explanation

The key finding here is ‘ the right part of the thorax is distended and takes no part in respiration’ this is an indicator for Tension Pneumothorax ( of the right side). Pneumothorax is the collapse of the lung when air accumulates between the parietal and visceral pleura inside the chest. A tension pneumothorax is a severe condition that results when air is trapped in the pleural space under positive pressure, displacing mediastinal structures, and compromising cardiopulmonary function ie, (distended neck veins, tracheal deviation, and hemodynamic instability)
98. A 55-year-old woman came to a gynecologist with complaints of leukorrhea and bloody discharge from the vagina after • 5 years of menopause. Anamnesis states no pregnancies. Bimanual examination: the uterus and uterine appendages are without changes. During diagnostic curettage of the uterine cavity the physician scraped off a sample of enchephaloid matter What is the most likely diagnosis in this case?

Explanation

99. What modern organizational method can provide the patients in the remote settlements with timely access to quality medical aid and such medical services as consulting, diagnostics, and treatment, especially in the situations when time and distance are crucial?

Explanation

Telemedicine is the exchange of medical information from one location to another using electronic communication, which improves patient health status. It involves remote diagnosis and treatment of patients by means of telecommunications technology.
100. A 17-year-old girl has been suffering from hepatic cirrhosis for 3 years. Lately her periods of excitation have been intermittent with depression, she does not sleep enough. Objectively her condition is severe, the girl is sluggish, gives one-word responses, has tremor in her extremities, her skin is icteric, with single hemorrhagic rashes. Name the likely complication of her disease:

Explanation

Recall that one of the major functions of the liver is detoxification i.e., converting harmful metabolites in the body into less harmful products that can be easily excreted out eg Ammonia is converted in the liver to urea which is then excreted via the urine. In  a case of hepatic cirrhosis, this function  of the liver is affected leading to the accumulation of harmful metabolites in the body system. Ammonia itself can easily cross the blood brain barrier causing encephalopathy. Notice that this patient presents with signs that suggest a reduction in mental activities. 

Reye syndrome is a rare form of acute encephalopathy and fatty infiltration of the liver that tends to occur after some acute viral infections, particularly when salicylates (aspirin) are used.
101. A man works in casting of nonferrous metals and alloys for 12 years. In the air of working area there was registered high content of heavy metals, carbon monoxide, and nitrogen. During periodic health examination the patient presents with asthenovegetative syndrome, sharp abdominal pains, constipations, pain in the hepatic area. In urine: aminolevulinic acid and coproporphyrin are detected. In blood: reticulocytosis, low hemoglobin level. Such intoxication is caused by:

Explanation

Aminolevulinic acid and coproporphyrin are amino acids necessary for the synthesis of Heme. The excretion of these constituents in urine indicates either an inherited or secondary Porphyria. One of the most frequent causes of the secondary form of this disease is Lead poisoning. An increased degradation of porphyrin will lead to a decrease in Hemoglobin synthesis. Also note that the patient experiences Asthenovegetative syndrome ( physical and nervous fatigue), pain in the hepatic area and stomach; these are classical signs of Lead poisoning.

 

Carbon monoxide poisoning will be characterised by headache, lethargy, dizziness, loss of consciousness etc.

102. During regular examination of a 2- year-old boy he presents with enlarged left kidney painless on palpation. The right kidney was undetectable on palpation. Excretory urography shows no contrast on the right. Cytoscopy detected hemiatrophy of the urinary bladder trigone, the right ureteral orifice is not detected. What pathology is it?

Explanation

 

An agenesis is the failure of an organ to develop. From the above analysis, we observe an undetected right kidney on palpation; an absent contrast on the right during excretory urography indicates no organ located ( supported by an incomplete bladder trigone) - coupled with the patient’s age, we can conclude that development of the right kidney never took place. A compensatory hypertrophy/ hyperplasia of the left kidney occurred in order to meet the body’s requirement.

103. A 34-year-old woman after rapidly changing her position from horizontal to vertical suddenly paled, fell down, her skin became moist, her limbs are cold, her pupils are dilated. The pulse is rapid and thready blood pressure is 50/25 mm Hg. What condition has likely developed in the patient?

Explanation

This patient just experienced an Orthostatic syncope or collapse. She was in the horizontal position (lying down) and when she rose to the vertical (upright position) she fell down. Orthostatic syncope refers to syncope resulting from a postural decrease in blood pressure. Classic orthostatic hypotension occurs when there is a persistent reduction in blood pressure of at least 20mmHg systolic or 10mmHg diastolic within 3 minutes of standing or being upright.

When the body assumes an upright position, there is an immediate gravitational pooling of about 500 to 1000ml of blood to the lower extremities, splanchnic and pulmonary circulations. The decrease in venous return to the heart reduces cardiac output and eventually causes a drop in blood pressure. Baroreceptors in the carotid and aortic arteries sense this decrease in blood pressure and activate the sympathetic nervous system which leads to increased heart rate, systemic vasoconstriction, and increased cardiac muscle contractility all of which eventually increase blood pressure.

 

In a healthy individual, this sympathetic activation causes a physiological increase in heart rate by 10 to 20 bpm, diastolic blood pressure by 5mmHg, but minimal to no change in systolic blood pressure. In patients with autonomic dysfunction, there is an inadequate engagement of the autonomic nervous system in response to a decrease in blood pressure leading to persistent hypotension and in serious conditions just as we have here, it can result in collapse.

104. On laboratory investigation of a pork sample there is 1 dead trichinella detected in 24 sections. This meat should be:

Explanation

This risk of getting Trichinellosis is very high if this meat is consumed; this piece should be technically disposed.
105. Employees work in conditions of high dust concentration. Certain chemical (silicon dioxide content) and physical properties of dust aerosols contribute to the development of occupational dust-induced diseases. What is the main physical property of dust aerosols?

Explanation

Dispersion is the separation or distribution of a given substance over a particular area. Dust is easily carried by and spread around by air.

Solubility refers to the ability of a given substance (the solute) to dissolve in another substance ( the solvent). 

 

Ionization refers to any process by which electrically neutral atoms or molecules are converted to electrically charged atoms or molecules (ions).

106. A woman complains of a severe pain in her throat on the left, difficult swallowing and mouth opening, elevated body temperature, and general malaise. The onset of the disease was 4 days ago after a case of tonsillitis. Examination detects a trismus of the masticatory muscles, the left tonsil is displaced toward the midline, the anterior palatal arch is infiltrated and protruding. The regional lymph nodes on the right are enlarged and painful on palpation. Make the diagnosis:

Explanation

A peritonsillar abscess is a bacterial infection that usually begins as a complication of untreated strep throat or tonsillitis (4 days ago, the patient had a case of tonsillitis). also known as quinsy, it leads to the  accumulation of pus due to an infection behind the tonsil. Key symptoms include; throat pain, difficulty swallowing , drooling , swelling of the face or neck, fever and chills. Notice that this patient presents with signs affecting the TONSILS; in a case of Pharyngitis, the posterior pharyngeal wall will be affected. In a case of infectious mononucleosis, Atypical lymphocytes will be present on analysis.
107. During winter epidemics of influenza caused predominantly by virus A/California/04/2009 (H1N1), on the 2nd day after the disease onset a 30-year-old hospitalized man presented with high fever, dry cough, myalgia, headache, and general weakness. What should be prescribed as an. etiotropic treatment in this case?

Explanation

An etiotropic treatment is one aimed at the causative agent. In this case a Virus - Influenza A. Oseltamivir is an antiviral drug aimed at the treatment of influenza A and B. It acts by inhibiting influenza neuraminidase ( an enzyme needed for influenza replication). Zanamavir also has this function.

 

Acyclovir is also an antiviral drug used mainly in treatment of the herpes Simplex virus and Varicella zoster virus.

108. A 30-year-old woman complains of itching skin, predominantly in the evening and at night. The condition lasts for 2 weeks already On the skin of the interdigital folds, mammary glands, abdomen, buttocks, and thighs there are numerous fine papular and papulovesicular rashes located in pairs,  excoriations. There is no rash on the face and neck. Similar rash is observed in the husband of the patient. What is the most likely diagnosis?

Explanation

Scabies is a parasitic skin infestation caused by the Sarcoptes scabiei. It is characterised by intense pruritus ( itching) at night; this characteristic and the location of the rashes helps us to diagnose scabies. The primary lesion found are usually papules, vesicles or burrows ( accompanied by excoriations and pustules) which are commonly seen in interdigital folds, flexor surfaces of the wrist, axillary folds, buttocks ( closed areas of the body). Additionally in children, elderly persons, and immunosuppressed patients: scalp, face, neck, under the nail, palms of hands, and soles of feet.

Neurodermatitis is also referred to as Lichen simplex chronicus; it is chracterised by chronic itching and scaling localised on the wrist, neck, forearm, legs and anal region. 

Epidermophytosis is referred to as ‘Athlete’s foot’.
109. A 20-year-old patient complains of a severe headache, double vision, weakness, fever; irritability Objectively: body temperature is 38.1°(7, the patient is reluctant to contact, sensitive to stimuli. There are ptosis of the left eyelid, exotropia, anisocoria S 〉 D , and pronounced meningeal syndrome. On lumbar puncture the cerebrospinal fluid flowed out under a pressure of 300 mm Hg, the fluid is clear, slightly opalescent. 24 hours later there appeared a fibrin film. Protein - 1.4 g/L, lymphocytes - 600 per mm3 , glucose - 0.3 mmol/L. What is the provisional diagnosis?

Explanation

Meningitis is an acute infectious disease with involvement of the arachnoid and pia of the brain and spinal cord by pathogenic microorganisms. Etiologically, the various forms include; bacterial, viral. Fungal and tuberculous meningitis. The tuberculous form occurs as a secondary infection; The CSF is under increased pressure, is clear or slightly cloudy, and contains a predominance of mononuclear cells (usually >400/mm3), increased protein (100 to 400 mg/dL), and a decreased glucose content.  Recall that a bacterial meningitis will have a predominance of leukocyte while a viral form will be predominated by lymphocytes in CSF count.
110. 3 hours after a trauma, a young man developed bradycardia of 46/min., anisocoria D>S, hemi-hyperreflexia S〉D, hemihypesthesia on the left, and a convulsive disorder The character of this process needs to be clarified. What method of examination will be the most accurate for this purpose?

Explanation

Notice that this patient had a trauma to the head from which he developed the signs of bradycardia, anisocoria, hemi-hyperreflexia, hemihyperesthesia and a convulsive disorder; a CT scan (computed tomography)  should be immediately carried out to get a clear image of the internal structures of the head to  find possible damages.
111. A dweller of the northern Dnieper area, a fisherman, for the last several days has been complaining of a discomfort in his right subcostal region, periodical episodes of diarrhea, intermittent with constipations, frequent skin rashes. Abdominal ultrasound shows enlarged liver and pancreatic head. Make the provisional diagnosis:

Explanation

From anamnesis, the patient is a fisherman by profession- with this we can assume that he consumes his primary product ( fish).  Opisthorchiasis is a trematode (fluke) infection caused by infection with one of the species of the liver fluke Opisthorchis, which is acquired by eating raw or undercooked freshwater fish containing infectious metacercariae. The three species are: O. sinensis (still widely known as Clonorchis sinensis), O. felineus/tenuicollis and O. viverrini. Mostly asymptomatic but can present with dyspepsia, abdominal pain, diarrhoea or constipation. In the long term, we begin to observe hepatomegaly and malnutrition. Trichinellosis is gotten after consumption of meat ( especially pork) infected by Trichinella spp. In this disease, the larva  enters into the bloodstream and encysts in striated muscles leading to myositis ( a key finding).

The adult form of Taenia solium and T. saginata cause taeniasis, T. solium larvae cause cysticercosis. They are differentiated by the number of proglottids/ uterine branches 7-12 for T. solium and 17-35 for saginata.

 
112. A 25-year-old woman was brought into the gynecological department with profuse bloody discharge from her genital tracts. She is 12 weeks pregnant, the pregnancy is planned. Within the last 3 days she was experiencing pains in her lower abdomen that eventually started resembling cramps, she developed bleeding. Her skin is pale, pulse - 88/min., blood pressure - 100/60 mm Hg, body temperature - 36.8°C. Vaginal examination: the uterus size corresponds to 11 weeks of gestation, the cervical canal allows inserting 1 finger and contains fragments of the fertilized ovum, the discharge is bloody and profuse. What is the most likely diagnosis?

Explanation

Spontaneous abortion refers to pregnancy loss at less than 20 weeks' gestation in the absence of elective medical or surgical measures to terminate the pregnancy. The types of spontaneous abortion include threatened, inevitable, incomplete, complete, septic, and missed abortion. A threatened abortion is defined as vaginal bleeding before 20 weeks gestational age in the setting of a positive urine and/or blood pregnancy test with a closed cervical os, without passage of products of conception and without evidence of a fetal or embryonic demise. Note that this is not a case of threatened abortion because we can observe fragments of the fertilised ovum accompanied by profuse and bloody discharge.
113. A 39-year-old man, a battery attendant, suddenly developed weakness, loss of appetite, nonlocalized colicky abdominal pains, and nausea. Objectively his skin is gray; there is a pink-gray stripe on his gums; the abdomen is soft and sharply painful. Blood test detected erythrocytes with basophilic stippling and anemia. The patient has a history of peptic ulcer disease of the stomach. Constipations occur each 3-4 days. What is the most likely provisional diagnosis?

Explanation

Lead inhibits ferrochelatase and ALA dehydratase which leads to a decrease in Heme synthesis and increased RBC protoporphyrin. Lead also inhibits rRNA degradation which results in RBC’s retaining aggregates of rRNA (seen as basophilic stippling). Symptoms of lead poisoning include; Lead lines on gingiva ( Burton lines) and on metaphysis of bones ( on x-ray), Encephalopathy and erythrocyte basophilic stippling, Abdominal colic and sideroblastic anemia, wrist and foot Drop. A chelating agent such as Dimercaprol is used as a first line of treatment.
114. An 18-year-old patient complains of skin rash. The patient has been suffering from this condition for 5 years. The first instance of this disease occurred after a car accident. Objectively: the patient presents with a papular rash covered in silvery scales, ’’thimble” sign (small pits on the nails), affected joints. What is the most likely diagnosis?

Explanation

Psoriasis is a common skin disorder characterised by the presence of papules and plaques with silvery scaling especially on the knees and elbows. Other characteristics include acanthosis, parakeratosis and pinpoint bleeding ( Auspitz sign). The patient presents with a thimble symptom which translates into a pitting or a psoriatic nail.

 

A Panaritium or Whitlow is an acute purulent inflammation of the tissues of the finger or toes. Onychomycosis is a fungal infection of the nails; signs are white or yellow nail discoloration, thickening of the nail, and separation of the nail from the nail bed; it is also known as  tinea unguium.

115. During regular medical examination a lyceum student presents with signs of cheilitis that manifests as epithelial maceration in the area of lip seal. The lips are bright-red, with single vertical cracks covered with brown-red scabs. These clinical signs are most likely caused by insufficient content of the following in the diet:

Explanation

The signs and symptoms of riboflavin (vitamin B2) deficiency typically include sore throat with redness and swelling of the mouth and throat mucosa, cheilosis and angular stomatitis (cracking of the lips and corners of the mouth), glossitis, seborrheic dermatitis or pseudo-syphilis, and a decreased red blood cell count with normal cell size and hemoglobin content (normochromic normocytic anemia).

 

Thiamine (vit B1) deficiency is known as Beri-beri, deficiency of ascorbic acid will lead to scurvy, easy bruising and bleeding are seen in such patients.

116. Children from a certain township present with brittle teeth, malocclusion, dental enamel erosions, and dental pigmentation that looks like yellow-brown spots. What is the likely cause of this presentation?

Explanation

Presence of dark-yellow pigmentation of the tooth enamel, diffuse osteoporosis of bone apparatus, ossification of ligaments and joints are specific symptoms that indicate Fluorosis. This is a condition that occurs as a result of excessive intake of fluorine.
117. An 8-day-old boy was delivered to the hospital on the second day after the onset of the disease, His parents complain of his fussiness, regurgitation, body temperature to 38.5°C, red skin with infiltration in lumbar area. His medical history has no peculiarities. The child is in the severe condition, inert, pale, suckles poorly In the lumbar area, on the sacrum and buttocks there is a tense infiltration with hyperemic and cyanotic areas and with a soft spot 8x7 cm in its center. The stool is 10 times in 24 hours, with green and mucous admixtures. What is the most likely diagnosis?

Explanation

This patient presents with obvious signs of inflammation with an infective focus in the lumbar area. In children of this age group, they can be prone to some infections as their immune system is not yet strong enough to ward off pathogenic organisms.

Phlegmon is a medical term describing an inflammation of soft tissue that spreads under the skin or inside the body. It’s usually caused by an infection and produces pus. The name phlegmon comes from the Greek word phlegmone, meaning inflammation or swelling. Skin phlegmon can be: red, sore, swollen and painful.

 

Erysipelas most often involves the lymphatics. It is a red, spreading, indurated demarcated lesion involving the lymphatics of the skin and caused by streptococci.

118. A 72-year-old man with pneumonia complains of marked dyspnea, chest pain, severe cough with expectoration, t° is 39.5-40°C, no urination for the last 24 hours. Objectively the patient is conscious. Respiratory rate is 36/min. Percussion sound is dull over the right lower pulmonary lobe; on auscultation there is bronchial respiration and numerous moist crackles. Blood pressure is 80/60 mm Hg, Heart rate is 120/min. Heart sounds are muffled, there is tachycardia. What tactics should the family doctor choose for the management of this patient?

Explanation

Observe that this patient presents pneumonia, high grade fever, low blood pressure and has  been unable to urinate for the past 24 hours. This patient should be admitted to the intensive care unit; a urinary catheter should be inserted into the urethra inorder to drain out the accumulated urine. After rectifying the issue of urinary flow, he can then be transferred to the pulmonology unit.
119. A 48-year-old man came to a doctor with complaints of vomiting that brings no relief and a burning pain in his left subcostal region that irradiates to the left lumbar region. These signs appeared after a meal. The Ortner’s and Mayo-Robson’s signs are positive. In the blood: leukocytosis and increased ESR. In the urine: elevated diastase levels. Make the diagnosis:

Explanation

Ortner’s sign- tenderness on light percussion on right costal margin by the edge of the palm

Mayo-Robson’s signs- pain while pressing at the top of the angle lateral to the erector spinae muscles and below the left 12th rib left costovertebral angle.

 

These signs are present in a patient with pancreatitis and can be confirmed by the elevation of Diastase level. Urine diastase is useful in diagnosing uncertain abdominal cases (especially when pancreatitis is suspected).

120. A 32-year-old woman complains of general weakness, low-grade fever persisting for 4 months, lumbar pain, and dysuria. Anamnesis includes frequent acute respiratory diseases, overexposure to cold, low-calorie diet, a case of pulmonary tuberculosis in childhood. Clinical urine analysis: pH- 4.8, leukocyturia, hematuria. Complete blood count: leukocytosis, lymphocytosis, raised ESR. Urography concludes: dilatation of renal pelvis and calyceal system of both kidneys, foci of calcification in the projection of right kidney parenchyma. What is the most likely diagnosis?

Explanation

Notice that from anamnesis, this patient already had a case of tuberculosis in childhood. Nephrotuberculosis ( renal tuberculosis ) refers to a disease of the kidney due to Mycobacterium tuberculosis, usually from bacillemia ( bacilli in circulation) in cases of pulmonary tuberculosis. Pathological changes include granulomatous inflammation and caseous necrosis of kidney tissue.
121. An unconscious patient was delivered to a hospital by an ambulance. Objectively his body temperature is 39°C, he presents with convulsions and red dry skin. It is known that the patient works as a stoker in the boiler room. What is the likely diagnosis?

Explanation

From anamnesis, we observe that this patient works in a very hot/ steaming environment; the signs shown by this patient ie. a red dry skin and a temperature of 39℃ indicates the effect of heat on the patient. Heat stroke is a form of hyperthermia in which the body temperature is elevated dramatically. The cause of heat stroke is an elevation in body temperature, often accompanied by dehydration.

 

The most common symptoms of CO (carbon monoxide) poisoning are headache, dizziness, weakness, upset stomach, vomiting, chest pain, and confusion on the background of inhalation of fumes from cars, generators etc.

122. A child was born at 40 weeks of gestation with the weight of 3700 g. The child’s Apgar score is 7/9. The baby was put to breast immediately after birth and suckled actively. On the 3rd day of life the child’s weight decreased to 3600 g. What transitory condition is observed in this child?

Explanation

Almost all newborns lose weight in the first days of life , and this is called physiological weight loss. It is mainly due to fluid reduction and the use of adipose tissue as a source of energy by the newborns.
123. 2 hours after eating unknown mushrooms, a 28-year-old man sensed a decrease in his mobility and deterioration of his ability to focus. This condition was then followed by a state of agitation and aggression. On examination he is disoriented and his speech is illegible. 4 hours later he developed fetor hepaticus and lost his consciousness. What syndrome can be observed in this patient?

Explanation

Fetor hepaticus is a strong musty smell observed from the breath of a patient whose liver is failing to detoxify a toxic substance. An acute liver failure is observed when the liver loses its function within days or weeks. Note that this patient started experiencing the expressed symptoms a few hours after consuming mushrooms; it can also be caused by infection, alcohol or drug induced. 

Cytolytic syndrome is a type of liver disease that leads in cell destruction; the major cause of this disorder is paracetamol overdose.

 

Hepatolienal syndrome involves the reduction of kidney function accompanied by liver cirrhosis or liver failure.

124. A 17-year-old girl complains of a pain in her knee and ankle joints and body temperature up to 39*C. 2 weeks ago she had a case of acute tonsillitis. Objectively, her joints are swollen, sharply painful, and their mobility is reduced. On the skin of her trunk and limbs there are barely visible circle-shaped pale pink spots. Heart rate is 95/min, blood pressure is 90/60 mm Hg, heart sounds are weakened, there is a soft systolic noise over the apex. Make the provisional diagnosis:

Explanation

 

Acute rheumatic fever typically occurs 2-3 weeks after Group A streptococcal pharyngitis. Diagnosis depends on a set of clinical signs that result from this infection ( Jones criteria)- these include chorea, carditis, subcutaneous nodules, erythema marginatum, and migratory polyarthritis. For biochemical analysis, certain key markers are used in diagnosis; these include C reactive protein ( an inflammatory marker), antistreptolysin O titer ( a marker that indicates a group A streptococcal infection ), ESR ( a marker from general blood analysis). Using these criterias, we can observe that this patient has arthritis, carditis and results from the biochemical analysis all appear positive for an acute rheumatic fever.

Reactive arthritis is also known as reiter’s syndrome  and refers to the inflammation of joints as a result of  a urogenital, intestinal or respiratory infection.

125. A 19-year-old girl complains of moderate itching and hair loss on her head. Objectively on the skin of her occipital region there is a single round erythematous focus 3 cm in diameter with clear margins. Asbestos-like scales can be observed on the surface of the lesion. The hair in the focus of the lesion is broken off at the length of 6-8 mm. What is the most likely diagnosis?

Explanation

The description - a single round erythematous focus 3cm in diameter, asbestos like scales accompanied by itching and hair loss on the affected area is typical for Ringworm (tinea capitis in this case) also known as Microsporai; an infection caused by microsporum fungi.

Scabies is a parasitic skin infestation caused by the Sarcoptes scabiei. It is characterised by intense pruritus ( itching) at night; this characteristic and the location of the rashes helps us to diagnose scabies. The primary lessions found are usually papules, vesicles or burrows ( accompanied by excoriations and pustules) which are commonly seen in interdigital folds, flexor surfaces of the wrist, axillary folds, buttocks ( closed areas of the body).
126. A woman complains of frequent watery stool (up to 9-10 times per day) with mucus and blood admixtures, dull pain in the hypogastrium, weight loss of 4 kg within the last year. Objectively: malnutrition, dry skin, low turgor, aphthous stomatitis. The abdomen is soft, the sigmoid colon is spastic and painful on palpation. Occult blood test is positive. Fibrocolonoscopy: edema, hyperemia, mucosal granulation, pseudopolyps, small ulcers with irregular edges. Make the diagnosis:

Explanation

  1. Distinguishing Ulcerative Colitis from Crohn Disease

Ulcerative Colitis

Crohn Disease

Only colon involved

Panintestinal

Continuous inflammation extending proximally

 from rectum

Skip-lesions with intervening normal mucosa

Inflammation in mucosa and submucosa only

Transmural inflammation

No granulomas

Noncaseating granulomas

Perinuclear ANCA (pANCA) positive

ASCA positive

Bleeding (common)

Bleeding (uncommon)

Fistulae (rare)

Fistulae (common)

Cathartic colon

Occult blood in stool is indicative of a hemorrhagic process from the colon, while the results from the colonoscopy indicates the pathology is located in the colon.

 

Irritable bowel syndrome is an inflammatory bowel disease characterised by recurrent abdominal cramps (pain), change in form (consistency) and frequency of stool, constipation etc. . This condition is common in middle aged women and is associated with underlying conditions such as stress, anxiety, depression or a previous case of intestinal infection.

127. A 9-year-old boy fell off a tree and hit the back of his head. A brief loss of consciousness was observed. The child\'s condition is satisfactory, he has a headache and vertigo. Skull X-ray scans show a depressed fracture of the occipital bone in the area of the external occipital protuberance. What treatment tactics is indicated for this patient?

Explanation

 

The results from the x-ray shows the fracture of the skull coupled with the fact that this patient had a brief loss of consciousness; from the listed options, an immediate surgical intervention should be carried out.

128. A 16-year-old boy developed dizziness. His heart rate is 35/min, blood pressure is 85/45mmHg. Heart borders are not enlarged. Heart sounds are loud and clear. ECG shows P waves disconnected from QRS complexes, dissociation and different rhythm of the atria and ventricles is accompanied by varying location of P wave in relation to QRST complex. This presentation is the most characteristic of the following disease:

Explanation

Key point here is: QRS complexes DISCONNECTED with P waves.

This is the only information you need to answer this question.

Now let’s compare the answer choices. I won’t bore you with ECG images to avoid confusions. 

 

Atrioventricular (AV) block is partial or complete interruption of impulse transmission from the atria to the ventricles. We have 3 degrees of heart block.

First Degree AV block: All normal P waves are followed by QRS complexes, but the PR interval is longer than normal (>0.20 second).

Second Degree AV block: Some normal P waves are followed by QRS complexes, but some are not. Two types exist:

  • Mobitz type I

  • Mobitz type II

In Mobitz type I 2nd-degree AV block, the PR interval progressively lengthens with each beat until the atrial impulse is not conducted and the QRS complex is dropped (Wenckebach phenomenon); AV nodal conduction resumes with the next beat, and the sequence is repeated.

In Mobitz type II 2nd-degree AV block, the PR interval remains constant. Beats are intermittently nonconducted and QRS complexes dropped, usually in a repeating cycle of every 3rd (3:1 block) or 4th (4:1 block) P wave.

Third Degree AV block: Heart block is complete in 3rd-degree AV block. There is no relationship between P waves and QRS complexes (AV dissociation) in 3rd-degree AV block. 

 

There is no relationship between P wave and QRS complexes, which also means P wave is UNCONNECTED with QRS complexes.

 
129. A 56-year-old woman has been working as a disinfector for 19 years. She complains of general weakness, nausea, bitter taste in her mouth, heavy sensation. In her right subcostal area, and rapid fatigability. Objectively, her body temperature is 37.1 C, the sclerae are icteric and the liver is enlarged. Total Bilirubin is 40 mcmol/L. What is the likely diagnosis?

Explanation

The liver is the main organ responsible for the metabolism of drugs and toxic chemicals, and so is the primary target organ for many organic solvents. Work activities with hepatotoxins exposures are numerous and, moreover, organic solvents are used in various industrial processes. Organic solvents used in different industrial processes may be associated with hepatotoxicity. 

Work activities with hepatotoxin exposure are numerous and include chemists, dry cleaners, farm workers, painters, health care workers, nurses, and printers. Exposure to hepatotoxins can occur through intentional or accidental ingestion in food or absorption of toxic contaminants through the skin. 

The patient in question who works as a disinfector for 19 years and presented with signs of liver pathology (liver is enlarged, jaundice) most probably works in the farming industry or health care setting. Farmers and many workers in the agro-food chain are exposed to these substances as well as consumers who eat agri-cultural products that are not properly cleaned and decontaminated.

 

Among these substances, 1,1,1-trichloro-2,2-bis (p-chlorophenyl)-ethane (DDT) and its metabolite 1,1-dichloro-2,2-bis (p-chlorophenyl)-ethylene (DDE) have been extensively studied. DDT was used both in agriculture and for environmental disinfection until its use was later forbidden in both America and Europe because of its toxic effects on humans. In humans, DDT contamination occurs through con-tact with the skin, mucous membranes and inha-lation.

130. A 43-year-old man complains of facial edema, dyspnea, and difficult swallowing that appeared 3 hours after he was stung by an insect. Objectively the skin of his eyelids, cheeks, and lips is acutely hyperemic and edematous. What medicines should he be prescribed for emergency aid?

Explanation

 

This patient is having an Anaphylactic reaction (Type I hypersensitivity reaction) after being stung by a bee. Notice that he is having inflammation of the airway, making it difficult to breath. For emergency care , this patient should be administered Intravenous epinephrine and glucocorticoids to reduce inflammation and help improve breathing.